Professional Responsibility Outline

87
INTRODUCTION: 6 Themes of Professional Responsibility 1. Conflicts of Interest a. Lawyers should have undivided loyalty to their clients. b. 1.7, 1.8, 1.9 (former clients) c. lawyer vs. client 2. Truthfulness a. Honesty: Rule 8.4 prohibits dishonesty, fraud, deceit and misrepresentation (Misconduct 8.1-Applicant) b. 3.3 Candor toward tribunal: You’re an officer of the court! 3. Lawyer’s duty to client vs. duty to system (spectrum of “client- centered” vs. “public-centered”-most lawyers are somewhere in the middle): Attention to the “public interest” (it’s not okay to just forward the interests of your client, you musts consider the public interest). a. Rule 1.4: Scope of representation 4. Lawyer’s personal/professional interests vs. fiduciary obligations: Attorneys have a unique position of TRUST 5. Self-interests (we are self-regulated- see comment number 12 in the preamble) a. We are self regulated, but how do we embrace our professional norms? b. Compare comment 12 in the preamble with Rule 1.5 (discretionary power over fees) which is ambiguous about attorney’s fees, notifying clients about billing. Ex: Fen-Phen case 6. Institutional Pressures on professional judgments (rookie attorneys poor judgment will not be excused because they are inexperienced. Supervising attorneys can be held responsible as well for poor judgment on behalf of new attorneys) a. 8.3: Reporting misconduct “Often, but not always, the right thing to do in a particular situation will also comply with the ethical rules.” Three questions to assess your conduct (when the right thing conflicts with the ethical thing to do): 1. Does the conduct in question violate the ethical rules? 2. Does it violate some other law (criminal, regulative?) (Along with the ethical rules, you have to worry about all other laws like the Constitution, statues, etc.). 3. What is the right thing to do? This isn’t always going to be determinative of your conduct though. CHAPTER 1: The Regulation of Lawyers INSTITUTIONS THAT REGULATE LAWYERS: 1. The highest state courts: a. State and local bars:

description

Professional Responsibility Outline aka Lawyer Ethics

Transcript of Professional Responsibility Outline

Page 1: Professional Responsibility Outline

INTRODUCTION:6 Themes of Professional Responsibility1. Conflicts of Interest

a. Lawyers should have undivided loyalty to their clients.b. 1.7, 1.8, 1.9 (former clients)c. lawyer vs. client

2. Truthfulnessa. Honesty: Rule 8.4 prohibits dishonesty, fraud, deceit and misrepresentation (Misconduct 8.1-Applicant)b. 3.3 Candor toward tribunal: You’re an officer of the court!

3. Lawyer’s duty to client vs. duty to system (spectrum of “client-centered” vs. “public-centered”-most lawyers are somewhere in the middle): Attention to the “public interest” (it’s not okay to just forward the interests of your client, you musts consider the public interest).

a. Rule 1.4: Scope of representation4. Lawyer’s personal/professional interests vs. fiduciary obligations: Attorneys have a unique position of

TRUST5. Self-interests (we are self-regulated- see comment number 12 in the preamble)

a. We are self regulated, but how do we embrace our professional norms? b. Compare comment 12 in the preamble with Rule 1.5 (discretionary power over fees) which is ambiguous

about attorney’s fees, notifying clients about billing. Ex: Fen-Phen case6. Institutional Pressures on professional judgments (rookie attorneys poor judgment will not be excused

because they are inexperienced. Supervising attorneys can be held responsible as well for poor judgment on behalf of new attorneys)

a. 8.3: Reporting misconduct

“Often, but not always, the right thing to do in a particular situation will also comply with the ethical rules.”

Three questions to assess your conduct (when the right thing conflicts with the ethical thing to do):1. Does the conduct in question violate the ethical rules?2. Does it violate some other law (criminal, regulative?) (Along with the ethical rules, you have to worry about all other laws like the Constitution, statues, etc.).3. What is the right thing to do? This isn’t always going to be determinative of your conduct though.

CHAPTER 1: The Regulation of Lawyers

INSTITUTIONS THAT REGULATE LAWYERS:1. The highest state courts:

a. State and local bars:b. We have an Integrated bar: Admission to the bar in Kentucky: KY SCR 2.000 et seq: Bar application

i. To be admitted, you must pass the MPRE (2.015), must have good moral character, must have approved bar application, etc.

c. Inherent powers doctrine: Courts have the exclusive authority to regulate the bar. Chambers v. Stengell talks about balancing the police power and the exclusive authority of the courts to regulate the bar.

d. How state courts regulate lawyers:i. adopts ethics codes and court procedural rules to govern lawyersii. sets and implements standards for licensing of lawyers, including educational and moral character

requirementiii.supervises agencies that investigate and prosecute complaints of unethical conduct by lawyersiv.supervises administrative judicial bodies that impose sanctions on lawyers who violate ethics codes

2. State and local bar associations:3. Lawyer disciplinary agencies: 4. ABA: a private nonprofit membership organization founded in 1878. State bar associations are

independent, not subordinate to the ABA.a. Model rules:

5. American Law Institute:6. Federal and state trial courts:7. Legislatures:

Page 2: Professional Responsibility Outline

8. Administrative Agencies:9. Prosecutors:10. Malpractice insurers:11. Law firms and other employers:12. Clients:

Elements of a lawyer-client relationship:1. Competence (1.1)2. Confidentiality (1.6)3. Agency (lawyer as agent of the client)4. Fiduciary: “unique position of trust and confidence”5. Undivided loyalty and diligence6. Duty to inform (Rule 1.4) communication

Fen-Phen Attorneys illustrate a systemic and personal dilemma

• Preamble:• 1, 9, 12, 1.4 (failure to communicate), 1.5, 3.3 Candor toward tribunal, 8.3 reporting, 8.4 misconduct

• CASES-BAR APPLICATIONS:• In re Prager: Int’l drug smuggling, fled, extradited, probated, but graduated summa cum laude and clerked for

state supreme court judge: Denied. Good acts insufficient to out balance 16 years of pot smoking and smuggling

• Vaughn v. Bd of B. Examiners: Sexual relations with students (applicant used to be a public schoolteacher): Denied, court said his ethical value system was “deplorable”

• Fla. Bd. of B. Examiners re S.M.D.: Declaration of bankruptcy after charging wedding, credit cards, etc. Admitted: record did not sufficiently demonstrate financial irresponsibility

• Application of Gahan: Declaration of bankruptcy after law firm he worked for could not pay him-discharged his 14,000 student loan: Court did not find moral turpitude or fraud, but denied admission because his hardship was no “compelling” and failure to repay loans showed lack of good moral character.

• In re Tobiga: Shoplifting of meat- many positive character witnesses: applicant found to have proven moral character.

• Friedman v. conn. B. Examiners: Student accused of cheating on law school exam. Denied. The student who claimed the cheating was more credible than the other student, and there was other evidence to suggest applicant lacked good character

• Radtke v. Bd. of B. Examiners: Plagiarism from a university teaching position was concealed, application denied. Allowed to re-apply after one year. The underlying misconduct was unconnected to any legal work.

• In re Converse: Pattern of hostile and disruptive conduct: nude caricatures, display of nude female’s backside in his study carrel, launched personal attacks. Denied.

• PROBLEM 1-1, POT: PAGE 57• During law school, you’ve smoked pot a few times. You need to disclose this. Taken from Iowa (most people

are admitted).• Rule 8.1: applicant shall not knowingly make a false statement of material fact.

• Cases illustrate how subjective the judgment is. It also illustrates how intrusive the bar admission is.

In re Converse In re Tobiga SMD

Bar admission denied for law student engaging in provocative acts (pattern of hostile conduct)

Shoplifting for stealing meat and lied about unpaid loans; admitted

financial responsibility case, admitted

• Mental health of applicants: • The mere fact of treatment for mental health problems or addictions is not, in itself, a basis on which an

applicant is ordinarily denied admission in most jurisdictions. Many states, until the 80s and 90s, asked very

Page 3: Professional Responsibility Outline

intrusive questions. Now most states have narrowed their questions to ask about mental illnesses that require hospitalization or that involved psychotic disorders.

• Rose Gower • She spent two weeks in the hospital for depression during high school. She disclosed on her bar application,

and then the bar committee delayed her admittance for more than a year with continuous questions requiring more information about her medical history. (Discuss balance between safeguarding against inappropriate character/fitness and candor)

• In re Mustafa: • Highest court in DC: Mustafa was a co-chief justice of the law school’s moot court program and he took

money out of the organization’s bank account. It was during his third-year in law school. The other co-chief justice turned him in and he also turned himself in that same day. He claimed he took money out to pay his sister’s bail and he did pay it back. Admission to bar denied because he failed to establish fitness.

• Mustafa later was admitted to practice in California, but was fired and sued for wrongful termination (clients claim he did not return phone calls, appear at hearings, used business funds for personal use, etc.)

• How was he able to practice in California after being denied admission in DC?• If something happens in law school, as opposed to earlier in one’s life, it’s regarded as more seriously as

indicative of bad moral character• “clear and convincing evidence that you possess good moral character”• How can we prevent future misconduct?• Improvements that could be made:

• Clients don’t know anything about the disciplinary system• the sanctions imposed are often light and innocent- private sanctions, which are the lightest form of

discipline, are imposed almost twice as often as other sanctions. They fail to address primary goal of lawyer discipline, protection of the public.

• Problem 1-2: The doctored resume: • Third-year law student from another country lied on her resume, changed transcripts, etc. You’re on the

Honor Council: Should you expel her, determine some other punishment, or feel bad for her and let her stay.• Options: Expel, violation of the honor code, warning (consensus: warning not stringent enough). She

probably needs to correct the resume and disclose it to the bar, and possibly a note in her law school file. • She can’t be penalized under the bar yet since she’s only a student- but she’ll likely be denied admission to

the state bar.• Naivity cannot be an excuse for lying.

CHAPTER 2: LAWYER LIABILITY: • Notion of self-reporting: Reporting is the key notion of self-regulation, but it can lead to under-

enforcement. You can be disciplined for conduct OUTSIDE of practicing law!1. Professional Discipline:

a. The history and process or lawyer discipline i. Process:

1. Complaint filed by a client or lawyer2. Bar counsel investigates complaint

a. If complaint does not warrant charge, filed closed, ORb. Charges filed if warranted by investigation

i. Hearing committee conducts hearing, makes factual findings, recommends sanctionsii. Hearing committee decision reviewed by judicial agency and/or by highest state court.

Reviewing body makes final decision on sanction.b. Grounds for discipline

i. Tens of thousands complaints received annually are not addressed because there are so many.ii. In re Peters: Dean of William Mitchell College of Law was reprimanded for engaging in unwelcome

physical contact and verbal communication of a sexual nature against four women employees. He was publicly reprimanded: case was made public and probably a notice was published in the bar journal announcing the decision. He wasn’t publicly reprimanded for five years. Was this enough? No...

1. A lawyer can be disciplined for actions which are illegal but do not result in criminal conviction

Page 4: Professional Responsibility Outline

2. Peters was eventually forced to resign from the law school but serves as president of an organization and was honored for his work with a charities organization a few years ago.

3. Deans and professors can be punished the same way lawyers are- they are still attorneys.iii.Common categories of questionable conduct by law professors:

1. sexual harassment2. plagiarism3. neglect of teaching responsibilities4. manipulation of grades5. aggressive or discriminatory behavior6. dishonest behavior

iv.Lawyers can be disciplined for violation of the ethics code whether or not the violation occurs in the course of law practice (any conduct that is dishonest or prejudicial to the administration of justice or that reflects lack of fitness to practice). Other conduct examples:

1. domestic violence2. failure to pay child support3. drunk driving4. even for putting slugs in parking meters!5. any crime that reflects dishonesty, untrustworthiness, or lack of fitness to practice

v. If a lawyer is disbarred or suspended in one state, and they are licensed in more than one state, they have to report it to all the states they are licensed to practice in.

vi.A lawyer can be disciplined for violating a rule in a state in which she is not licensed to practice. vii. A lawyer can be disciplined for something she does outside the state in which she is licensed to

practiceviii. A lawyer can be disciplined based on the actions of an employee.

←c. Reporting misconduct by other lawyers

i. The duty to report misconduct 1. Rule 8.3: a lawyer who knows that another lawyer has committed a violation of the rules of

professional conduct that raises a substantial question as to that lawyer’s honesty, trustworthiness or fitness as a lawyer in other respects, shall inform the appropriate professional authority...

a. A lawyer who “knows” of a violation by any other lawyer (an adversary, a partner, a boss, public official, another lawyer in their firm) must report it to the bar disciplinary agency

i. Exceptions: only those raising a “substantial question” of the lawyer’s honesty, trustworthiness or fitness” must be reported.

ii. “know”: see rule 1.0: “actual knowledge”b. This includes reporting misconduct by judgesc. does NOT require disclosure of information otherwise protected by Rule 1.6 (confidentiality

rules) or info gained while participating in an approved lawyers assistance programd. The duty to report is triggered by knowledge of another lawyer’s misconduct.e. Standard for assessing knowledge is objective. The knowledge must be more than a mere

suspicion that misconduct has occurred. Question is whether a “reasonable lawyer in the circumstances would have a firm opinion that the conduct in question more likely than not occurred”

f. Do not have to report confidential client information under Rule 1.6i. You could disclose if it wouldn’t hurt the client and they give informed consent

g. If you learn of misconduct during proceeding, you can wait until proceeding has concluded to protect a client’s interests.

h. This includes the duty to blow the whistle on your boss if she does something unethical. i. A lawyer may be subject to discipline if she fails to report serious misconduct by another

lawyer. Although there are few public reports of this happening, the possibility of discipline for not reporting has motivated many lawyers to come forward with reports of unethical behavior.

j. Daryl van Duch, Best Snitches: Land of Lincoln Leads the Nation in Attorneys Turning in Their Peers: (breaks down “Conspiracy of silence”) Illinois has staggering numbers of reporting (in 1995, nearly 600 complaints filed). Why? In 1988 the Ill highest court suspended an atty’s license for failing to report another lawyer’s misconduct. Facts: James Himmel helped client recover monies pocketed by first attorney, and never reported the first atty’s misconduct (the

Page 5: Professional Responsibility Outline

client asked him not to report it). He had a duty to report the attorney’s wrongdoing and was therefore disciplined.

k. The “Himmel Rule” also had negative repercussions- state had to make it unethical to threaten to snitch during pending civil litigation about opposing counsel’s alleged misconduct b/c too many lawyers were using skeletons to gain negotiation leverage.

l. Normally, in these situations you would not have to tell- here, the situation had been so public that the atty-client privilege was waived.

m.Some states like NY have adopted a “law firm rule” to hold entire firms accountable for misconduct.

ii. Lawyers’ responsibility for ethical misconduct by colleagues and superiors: (the ethics codes impose a limited amount of collective responsibility on other lawyers in the firm organization for the conduct of other lawyer and of non-lawyer employees)

1. Rule 5.1: explains responsibility of a partner or supervising atty for ensuring compliance with the ethical rules by subordinate lawyers, and explains when a senior lawyer may be subject to discipline for the conduct of a subordinate lawyer

a. Rule 5.1(a): A partner in a law firm, and a lawyer to individually or together with other lawyers possesses comparable managerial authority in a law firm shall make reasonable efforts to ensure that the firm has in effect measures giving reasonable assurance that all lawyers in the firm conform to the Rules of Professional Conduct

i. Lawyer managers must set up systems to prevent ethical problems (includes procedures to check for conflicts of interest and to manage client funds, providing continuing education in legal ethics). “law firm” includes legal services org’s or legal dept’s of corps, gvt agencies, etc.

b. Rule 5.1(b): A lawyer having direct supervisory authority over another lawyer shall make reasonable efforts to ensure that the other lawyer conforms to the Rules of Professional conduct

i. a supervising atty is not responsible for ethical violation of subordinate atty if they do not know about it. but the violation could be a breach of their supervisor’s duty under 5.1(b) if they don’t make reasonable efforts to prevent the violation

ii. SEE COMMENT 5 for Rule 5.1, Page 337 of Rules Supplementc. Rule 5.1(c): A lawyer shall be responsible for another lawyer’s violation if:

i. the lawyer orders or, with knowledge of the specific conduct, ratifies the conduct involved; or

ii. the lawyer is a partner or has comparable managerial authority in the law firm in which the other lawyer practices, or has direct supervisory authority over the other lawyer, and knows of the conduct at a time when its consequences can be avoided or mitigated but fails to take remedial action

2. Rule 5.2: explains when a subordinate lawyer is responsible for her own conduct, and when she can follow orders without fear or discipline

a. Rule 5.2(a): A lawyer is bound by the Rules of Prof. Conduct notwithstanding that the lawyer acted at the direction of another person

b. You cannot say, “I was ordered to do it, that’s why I did it!”. c. Rule 5.2(b): A subordinate lawyer does not violate the Rules of Prof Conduct if that lawyer acts

in accordance with a supervisory lawyer’s reasonable resolution of an arguable question of professional duty.

i. To know if the supervisor’s instruction is a “reasonable resolution of an arguable question of professional duty”, they should 1) do some research and 2) seek advice from someone more experienced.

ii. SEE COMMENT 2 ON PAGE 341 OF RULES SUPPLEMENT1. Someone has to make a judgment so when the supervisor does it, and turns out to be

wrong, the subordinate is safe from disciplined. In most states, only individuals can be disciplined for violations. But in NY and NJ, there can

be discipline on law firms for certain misconduct such as failure to supervise attorneyse. Problem 2-1: The Little Hearing: New associate thrown into a hearing for immigration law with

no experience and little time to prepare. Page 109. Rule 5.2b applies b/c her supervisor might be the one that gets in trouble. 1.1 is also relevant: Competence. See Rule 1.1, Comment 1 and 5 (page 27 of Rules Supplement). Rule 5.1b also applicable.

Page 6: Professional Responsibility Outline

i. Comment 1: factors for knowledge and skill:1. complexity/specialized nature of the matter2. lawyer’s general experience3. training and experience in the field in question4. preparation and study the lawyer is able to give the matter and whether it is feasible to

refer to matter to, associate or consult with, a lawyer of established competence in the field in question

ii. Comment 5: Thoroughness and preparation factors:1. inquiry into and analysis of the factual and legal elements or the problem2. use of methods and procedures meeting the standards of competent practitioners3. adequate preparation4. what is at stake

iii.she should either quit or report. Powell says there is a duty to reportiv.8.3 Integrity of the Profession: v. Rule 1.3 Diligence: “A lawyer shall act with reasonable diligence and promptness in

representing a client.”3. Rule 5.3 : (language almost identical to 5.1) explains responsibilities of lawyers who supervise non-

lawyer employees for ensuring they comply with rules of professional conduct.4. Wieder case (NY 1992) : law firms cannot dismiss attorneys for reporting misconduct. The

requirement that lawyers with evidence casting substantial doubt on another lawyer’s honesty, trustworthiness or fitness inform disciplinary authorities is an implicit part of his employment contract with the law firm. Statute: When a firm discharges a lawyer for making such a report, that firm can be sued for breach of contract. Until this case, the law provided no such protection and associates could be fired for any reason other than one expressly prohibited by law (like race).

5. Jacobson case (Illinois): Contrary case to Weider: Illinois Court ruled “there was no need to allow associates who were fired for insisting on compliance with ethical rules to be sue law firms for wrongful discharge.” Sometimes it’s economically advantageous to keep quiet.

6. Scott McKay Wolas: Kelly v. Hunton and Williams: TX: Kelly was a new associate at Hunton and Williams. He seemed to do well and he had good reviews from partners. He noticed that one of the partners; Scott Wolas was misreporting his hours so he tried to tell other partners about it. One of the partners told one of the associates to correct the misreported hours of Wolas. He received bad reviews after he started reporting it. The firm conducts a sham investigation, and finds no fraud. The associates want to report it, but the partners don’t think it should be reported. Kelly is encouraged to resign and does so. Wolas ended up disappearing and it turns out he was running a money scheme from his office and stole 30-40 million from clients. Court turned to the Wieder case for ruling. The firm tried to argue that because Kelly was not an admitted attorney yet, Wieder didn’t apply-

a. States differ. In Texas, they can fire you. In NY they can’t.i. See Rule 8.1 Applicants

7. The Photographer Problem 2.2 , page 126: Junior partner finds out that senior partner is overbilling big clients. He tells another senior partner, who recommends the junior partner report the overbilling. There is no statute on retaliatory discharge, and the firm’s partnership agreement doesn’t have any explicit language on whistleblowing. Anytime someone says anything, they’re let go. What do you do?

i. This was a real case in texas: the junior partner reported, and the middle partner who she reported to, passed it along to the senior misconduct partner. The junior partner’s work was cut, and she was told to look for work elsewhere. She sued for breach of fiduciary duty, and breach of contract (constructive ousting from the firm). She also files a complaint with the bar association. the bar council starts to investigate but the bar complaint is closed b/c client refuses to participate. In the civil case still going on, junior partner got compensatory and punitive damages for 4 mill for breach of fid duty (reduced later to 237,000). this is later overturned.

1. Texas supreme court: The firm did not owe the partner a duty not to expel her for reporting their overbilling. If you report, you run the risk of being fired. This decision is really scrutinized. Firm eventually went out of business.

2. It seemed like the TX court was giving deference to the harmony of partnership, trying to keep relations smooth between all partners (so this goes against whistleblowing)

Page 7: Professional Responsibility Outline

ii. Rule 8.3: Reporting misconduct:iii.The Kentucky 8.3: A lawyer acting in good faith when he incorrectly reports misconduct is

immune from charges, civil or criminal.iv.Rule 1.5 Reasonable Fees:

8. Rule 8.4(c): It is professional misconduct for a lawyer to engage in conduct involving dishonesty, fraud, deceit or misrepresentation

←2. Civil liability of lawyers:

a. Legal malpractice: We use the general term “legal malpractice” but it encompasses tort claims, breach of K, breach of fiduciary duty claims, etc. They all fall within the umbrella of legal malpractice.

i. Tort claim of legal malpractice: The client must assert:1. Lawyer owed a duty2. Lawyer failed to exercise competence and diligence normally exercised by lawyers in a similar

circumstance3. The breach of duty caused harm to plaintiff4. The client must show that “but for” the lawyer’s conduct, the harm would not have occurred (he

would have prevailed in the matter in which the lawyer represented him) (Case within a case)5. Most common mistakes that lead to malpractice liability:

• Ignores conflicts of interest• Sue former client for unpaid fee• Accept any client and any matter that comes along• “Do business” with your client• Practice outside your area of expertise

b. Malpractice insurance i. Lawyers are not required to carry malpractice insurance but a prudent lawyer should do so.ii. In many states, lawyers are required to disclose whether they have malpractice insurance either to the bar

or to clients directly. This information would then be available to the public.iii.What kinds of acts are unlikely to be covered by a malpractice insurance policy?

1. intentional acts including fraud, or other dishonest conduct like billing fraud2. orders of restitution of legal fees, fines, or penalties3. orders to pay punitive damages (some policies allow separate purchase for this)4. conduct of lawyers in other roles like a notary public, title agent, trustee, fiduciary5. Intra-firm disputes, like liability to former partners or former employees.

←c. Other civil liabilities of lawyers

i. Advising or assisting unlawful client conductii. Stealing iii.Lyingiv.Intentional infliction of emotional distressv. Violation of regulatory statutesvi.Breach of contractvii. Remember, lawyers are bound by law as are ordinary citizens

d. Disqualification for conflicts of interest i. In deciding motions to disqualify lawyers b/c of alleged conflicts, court do not simply interpret the ethics

code (these standards articulate rules for discipline, not disqualification). Many courts follow their own common law standards, which may not be the same as the ethics code.

3. Criminal liability of lawyers a. Lawyers are not protected from criminal prosecution for white-collar crime

4. Client protection funds a. Client protection funds are state-sponsored programs designed to reimburse clients whose lawyers have

stolen their money. Most programs are now called “Lawyers’ Fund for Client Protection”.b. They exist in every state

Page 8: Professional Responsibility Outline

c. States do this because in cases of intentional misconduct, malpractice insurance will not cover costs and clients do not get their money.

5. Summing up: What governs lawyers?

Moral Certainty: “I’m doing the right thing”. Do not assume certainty in resolving problems. You want to guard against absolutism, and always keep relativism in mind. Morality is what WE believe as right/wrong as people. Ethics is what the system says is wrong:

Chapter 3: The Duty to Protect Client Confidences

1. The basic principle of Confidentiality • Rule 1.6: (a) lawyer shall not reveal information relating to the representation of a client unless the client

gives informed consent, or the disclosure is impliedly authorized. (b) The lawyer may disclose for these 6 exceptions:

• To prevent reasonably certain death or substantial bodily harm• To prevent client from committing a crime or fraud that is reasonably certain to result in substantial

injury to the financial interests or property of another and in furtherance of which the client has used or is using the lawyer’s services

• To prevent, mitigate, or rectify substantial injury to the financial interests or property of another that is reasonably certain to result or has resulted from the client’s commission of a crime or fraud in furtherance of which the client has used the lawyer’s services

• To sure legal advice about the lawyer’s compliance with the rules• To establish a claim or defense on behalf of lawyer in a controversy b/w lawyer and client, to

establish a defense to a criminal charge or civil claim against the lawyer based upon conduct in which the client was involved, or to respond to allegations in any proceeding concerning the lawyer’s representation of the client

• To comply with a law or court order• “Shall not”: mandatory, defines proper conduct for purposes of professional discipline. (Comment 14 of

Scope)• “May”: permissive and the lawyer has discretion to exercise professional judgment• If disclosing under one of the 6 exceptions, the lawyer should first seek to persuade the client to take

suitable action to obviate the need for disclosure. The lawyer should not make disclosure adverse to client’s interest that is greater than reasonably believed to be necessary to accomplish the purpose. The disclosure is made in connection with judicial proceeding, it should be made in a manner that limits access to the info. Under the 6 exceptions, a lawyer is permitted, but not required to disclose information relating to a client’s representation to accomplish the purpose of the exception.

• Comments are important to reada. Protection of “information relating to the representation of a client”:

• Information that is protected as confidential:• All info relating to the matter on which the lawyer is representing the client, except information that

“generally known”• Personal information relating to the client that the client would not want disclosed• Information learned from the client, and information learned from interviews, documents,

photographs, observation, or other sources• Info acquired before the representation begins (like during preliminary consultation) and after the

representation terminates• Duty of loyalty survives the attorney-client relationship

• Notes or memoranda that the lawyer creates relating to the matter• Punishments for failing to protect confidentiality:

Page 9: Professional Responsibility Outline

• Professional discipline• Tort liability or contract for breach of duty• Disqualified from representation of one or more clients, or• Enjoined by a court from further revelation

• General discussion of work is fine, with no identifying or personal information.• The fact that you are representing a certain client is not confidential, unless the client does not want

anyone to know• ***Protection of client confidentiality also applies to disclosures by a lawyer that do not in themselves

reveal protected information but could reasonable lead to the discovery of such information by a third person. See cmt 4 for Rule 1.6

• Social interaction that reveals confidential client info is not allowed• The Restatement draws line between disclosures that could harm a client and those that could not. R says

whether there is a reasonable prospect of harm to a client depends on “whether a lawyer of reasonable caution, considering only the client’s objectives, would regard use or disclosure in the circumstances as creating an unreasonable risk of adverse effect either to those objectives or other interests of the client.”

• Dinner with Anna Problem 3-1: Can you tell your friend about the case you’re currently working on?• You disclosed basic facts of the case, client’s first name, name of the bar where incident took place,

name of the police officer, opinions about the case, plan of investigation, discovery/litigation. Is this too much to disclose?

• Yes, she could reasonably go and find out confidential info now (cmt 4)• It’s not good enough to swear her to secrecy: think about the people sitting around you.• It doesn’t stop you from speaking broadly about work though.

• Comment 4 to Rule 1.6: The rule prohibits a lawyer from revealing information relating to the representation of a client. This also applies to disclosures by a lawyer that do not in themselves reveal protected information, but could reasonably lead to the discovery of such information by a third person. The use of a hypothetical to discuss issues relating to the representation is permissible so long as there is no reasonable likelihood that the listener will be able to ascertain the identity of the client or the situation involved.

• Dinner with Anna Scene 2: Problem 3-2: • What if friend asks detailed questions and you answer them?• Well, some of what you say may be violations if you answer specifically. In this case, lawyer may

have crossed the 1.6 barrier because she gave a lot of detailed answers. Look to see whether one of the exceptions to 1.6 applies. Also, pg 160 says that a lawyer may talk if they are virtually certain that the listeners could not ascertain the identity of the client or if the lawyer could show that there is no “reasonable prospect” that the discussion would adversely affect the client. This is closer to the norm than the statutory language.

• You probably went too far- name of bar, name of client, • The permissive standard of the Restatement bars revelation of client information if there is a

“reasonable prospect” of harm to the client.b. Protection of information if there is a reasonable prospect of harm to a client’s interests

i. Unlike the Model Rules, the Restatement (Third) of the Law Governing Lawyers prohibits revelation of confidential information only if “there is a reasonable prospect that doing so will adversely affect a material interest of the client or if the client has instructed the lawyer not to use or disclose such information.” This allows more room for disclosure than ABA Model Rule 1.6 which prohibits disclosure of any such information except as permitted by the rules.

ii. The exercise of discretion and how you communicate is going to vary from lawyer to lawyer, so it’s hard to establish a black-letter law.

c. The bottom line i. Quote from Hazard and Hodes: the line of permissible and impermissible disclosure should be drawn at

“anonymity.” The lawyer should exercise self-restraint and resolve marginal cases in favor of non-disclosure.

2. Exceptions to the duty to protect confidences: a. Revelation of past criminal conduct

Page 10: Professional Responsibility Outline

i. if the crime is over, the lawyer can’t benefit from disclosing- the community however can benefit. The lawyer should protect as confidential most information about past criminal activity by clients- if it’s in the past, you do not disclose unless disclosure is reasonably necessary to prevent reasonably certain death or substantial bodily harm.

1. Reasonably certain: it will be suffered imminently or there is a present and substantial threat that a person will suffer such harm at a later date if the lawyer fails to take action to eliminate the threat.

2. Ex: a lawyer knows that a client has accidentally discharged toxic waste into town’s water supply may reveal this info to authorities if there is a present and substantial risk that a person who drinks the water will contract a life-threatening or debilitating disease and the lawyer’s disclosure is necessary to eliminate the threat or reduce the number of victims.

ii. Other exceptions:1. Client gives informed consent2. The disclosure is impliedly authorized3. Or disclosure is permitted by 1.6(b):

a. To the extent the lawyer reasonably believes necessaryb. To prevent reasonably certain death of substantial harmc. To prevent the client from committing a crime or fraud that is reasonably certain to result in

substantial injury to the financial interests or property of another and in furtherance of which the client has used or is using the lawyer’s services

d. To prevent, mitigate, or rectify substantial injury to the financial interests or property of another that is reasonable certain to result or has resulted from client’s commission of crime or fraud in furtherance of which the client has used lawyer’s services

e. To secure legal advice about complying with MRulesf. To establish a claim or defense against a client on behalf of lawyer in controversy against

client, to establish a defense to criminal charge or civil claim against lawyer based on their conduct in which the lawyer was involved

g. To comply with other law or court order.4. If a lawyer needs to reveal confidential information to protect her own interests, she must take steps

to avoid the need for revelation, to limit its scope, or to limit the dissemination of the information.iii.The Missing Persons Problem 3.3: Your defendant reveals during hypnosis that he killed other people in

the past. You’re not sure if he’s telling the truth- what do you do?a. Robert Garrow (murderer)b. Alert policec. Do a site investigationd. Do nothing about the missing girls

• Probably best to do nothing- In the real case, the attorney did check out the bodies but did not reveal the information: but you shouldn’t do anything- if you don’t go, you don’t know.

• 1.6b1, and 1.6b6: look at comment 15, the rule does NOT compel disclosure.• What about under the exceptions:

• complying with law (must report discovering of body): It’s permissive, so you still don’t have to disclose

• to prevent future harm/injury: again, it’s permissive, plus, these bodies are not in danger. they’re already dead.

• the attorneys did not get in trouble for not reporting, but the court emphasized that “an attorney must protect his client’s interests, but also must observe basic human standards of decency.”

• Problem 3-4: Missing Persons Scene 2: • You actually go to the site, take photos of body. What if one of the deceased’s parents comes to visit

you? Do you tell them you know about their daughter’s death and know where her body is?• Rule 1.6 Comment 18: You are held to confidentiality standards even after the lawyer client

relationship terminates• Real case: atty did not tell anyone and did not want prosecutor to know because it would add on

charges- closer to trial, prosecutor tried to sue for obstruction of justice but court ruled for attorney.• People v. Belge: real case suing the lawyer after he failed to reveal location of bodies.• You should have never let the parents in the office.

Page 11: Professional Responsibility Outline

• Problem 3-5 : if he escapes, and he once told you where he used to hide out, and he might have been stalking your children. Do you tell? You must use your discretion. Disclosure in this case would be permissible. In an instance like this, you should use your discretion, which may be why the rule is drafted that way.

• may you tell? yes, if you think the risk to others is great. Rule 1.6a says normally there is a duty but there is an exception under 1.6b

• Real life, attorney did tell- gun fight ensued and police killed suspect.b. The risk of future injury or death • Spaulding (P) v. Zimmerman (D)

• 1962: no 1.6 rule yet• Zimmerman was driving and Spalding was passenger, they wrecked and Spalding sued Zimmerman for

injuries. Zimmerman’s doctor examined Spalding and found an aneurysm, but they didn’t tell plaintiffs this (and plaintiff’s doctor did not discover it during the initial examination). They settled for not much money, and later the aneurysm was discovered.

• Canon said it was duty of a lawyer to protect client’s confidences• The settlement is vacated by the State supreme court. • Based on the modern rule 1.6, you could disclose• Under the old ABA Canons of Professional Ethics, there was an ambiguity of what confidentiality was-

You could try to find a way to disclose- talk to the doctor about telling the plaintiff, or talk to dr. to convince the defendant into disclosing...

• RULE 3.3: You have a duty to exercise candor toward the tribunal• Dinner with Anna Scene 3: Problem 3-7: You’ve been doing research on Diet Kola- there’s slight statistical

evidence to show it causes miscarriages and higher rates of limb deformities in their offspring... Your firm is working in the interest of the food company- can you tell your pregnant friend who drinks Diet Kola?• Say that it’s not good for pregnant women to drink caffeine and consume food items that have artificial

flavor• Rule 1.6a: can’t reveal info relating to representation of client (you’re lobbying for the food company)

unless they give consent (which they won’t). • This doesn’t fall into any of the 6 exceptions of 1.6b.• 1.6 appears to require that you do NOT disclose, but you would try to find a way to say little...tell her

little, tell her you can’t say anything else, tell her, but maybe not say anything. • It’s not reasonably certain death or substantial bodily harm.• Look at 1.6a particularly: maybe do NOT disclose• Ramifications of choice: You have to make a decision about the way you will handle these situations and

still have the impact you desire, along with upholding your PR.• Comment 6: Disclosure adverse to client: • COMPARE this with the brutality case: at some level you’re at risk of violating the rule and NOT

violating the rule. • Maybe this is an excuse in the rules. • What if your job is on the line? Conflict b/w practical reality of being fired and ethical duty to disclose...• Moral v. ethical (morals: your own system, ethics: system’s rules)

←c. Client frauds and crimes that cause financial harm:• lawyers should not facilitate in their clients commission of fraud• Rule 1.2(d): a lawyer shall not counsel a client to engage or assist a client in conduct that the lawyer knows

is criminal or fraudulent.• Comment 10: The lawyer is required to avoid assisting the client (either by engaging in acts or suggesting

how the wrongdoing might be concealed). If lawyer discovers client conduct after they take the case, they must withdraw.

• What conduct constitutes “assisting a fraud”:• Failure to disclose the fact that a lawyer has offered false testimony (court interpretation of Rule 3.3:

candor to tribunals).

Page 12: Professional Responsibility Outline

• Rule 1.0(d): Fraud: conduct that is fraudulent under the substantive or procedural law or the applicable jurisdiction and has a purpose to deceive.

• the drafters did not intend to include in this definition “merely negligent misrepresentation or negligent failure to apprise another of relevant information. For purposes of this rule it is not necessary that anyone has suffered damages or relied on the misrepresentation or failure to inform.”

• Rule 1.16(a): Sometimes withdrawal alone is insufficient: it may be necessary for lawyer to give notice of the fact of withdrawal AND to disaffirm any opinion, document, affirmation or the like.

• Rule 1.6b2: Confidentiality can be broken to prevent before conduct• Rule 1.6b3: Confidentiality after conduct has occurred.• Drafters of the new exceptions allowing revelation of financial crimes and frauds tried to balance three

policies:• Encourage frank communication b/w clients and lawyers• to prevent harm to the public• protect the “integrity of the profession” by allowing lawyers to be whistleblowers if their own work is being

used to commit crimes or fraud.• Rule 4.1(b): bars a lawyer from knowingly failing to disclose a nonconfidential material fact when

disclosure is necessary to avoid assisting a client’s fraudulent act.• Comment 1: misrepresentation can also occur by partially true but misleading statements or omissions that

are the equivalent of affirmative false statements.• Enron and Sarbanes-Oxley Act:

• 1.6b2-3 came about because of Enron...• Congress passed the act to prevent further episodes of massive corporate fraud. It authorized the SEC to

promulgate tough new disclosure rules for professionals, including lawyers, who practice before the Commission and become aware of clients’ frauds.

• Lawyers who practice before the SEC or who advise companies regulated by the SEC must report any information about securities fraud to the highest officials of the corporation

• Relationship b/w Rule 1.6-b-2 and 3, and the Sarbanes-Oxley Act: MR apply more broadly (not restricted to the disclosure of frauds by publicly held companies, nor to lawyers that practice before the SEC)

• The Sarbanes-Oxley Act could trump state PR, and may REQUIRE disclosure in some cases where revelation would not be permitted under MR 1.6 (b/c federal law preempts state law!)

• Reese’s Leases: Problem 3-7: client is forging documents, copying signatures, but then you talk to them and the client promises to stop doing it. What do you do?

• 1.6b3: Reveal the fraud?• Is revealing the fraud enough? Do we need to do something more?• Not revealing and continue to represent: Probably could NOT do that• What you should do: You should definitely reveal: presume continuing fraud in a case like this and think that

4.1 mandates disclosure. No matter what, you have to stop representing the client. • Not reveal and stop representing them?:

• 1.6 was not triggered for them b/c the fraud was in the past. Key here is that revelation is discretionary.• 4.1 comment 3: Mandatory, so no discretion• 1.16

• The real case: consulted an expert• had no duty to withdraw opinion letters they sent to the bank b/c at that time they were unaware• eventually this firm was replaced by another firm, and the new firm was not alerted of any ongoing

fraud- later, the old firm was sued and had to pay damages. the consultant who gave advice (a dean) did not get in trouble.

• How do we balance 1.2, 1.6, and 4.1?• Rule 1.6 is permissible, but 4.1 (truthfulness to others) makes revealing a requirement.• 4.1 actually serves a limit on the permissive disclosures of 1.6 • 4.1 is a subset of 1.6, this is a drafting problem.

• Other relevant rules in revelation of criminal or fraudulent conduct:

Page 13: Professional Responsibility Outline

• 8.4 general dishonesty...• 3.3 candor toward the tribunal• 1.13: duties of a lawyer representing an organization to call attention to crimes and frauds (when

someone associated with the business is acting or intends to violate the law)• 3.4: prohibits lawyers from hiding or destroying evidence or advising a witness to testify falsely.• 1.16-a: requires withdrawal from representation if continued representation would result in violation of

the rule• 1.16-b: permits withdrawal from representing a client who persists in criminal fraudulent conduct

• Other ethics rules besides 1.2, 1.6, and 4.1 that allow or require revelation of criminal or• Lawyers are NOT protected from civil or criminal liability if they elect not to reveal.• The exception to 4.1 is the 1.6 exceptions: 1.6 is a subset of 4.1• Problem 3-10: the investment project

• Is it ok to buy the last piece of property when you know your client is not going to buy it?• Maybe they would change their mind so you should be cautious• Cmt 5 to 1.8: when you use info to client’s disadvantage, it violates your loyalty; the rule does NOT

apply to uses, which do not disadvantage your client.• If they know they’re not going to buy it, you can go ahead and purchase it. • If they were still considering it, you would need informed consent. Here you don’t need it, but you

should be safe and get it. • We could buy this land because we are not disadvantaging the client, because the client decided not to buy

the land• Rule 1.8(b): lawyer shall not use information relating to representation of a client to the disadvantage of the

client unless the client gives informed consent, except as permitted by rules• Comment 5: rule 1.8-b applies when the information is used to benefit either the lawyer or a third person,

such as another client or business associate of the lawyer-rule does not prohibit uses that do not disadvantage the client.

• For example, you learn a gvt agency’s interpretation of trade legislation during the representation of one client and you can properly use that info to benefit other clients.

• It’s not a good idea to go into business with a client for several reasons:

• Problem 3-9: Rat Poison: • Your client is being charged with manslaughter for putting rat poison in home where child ate it and died. he

keeps slacking on going to the other homes where he used the same poison to remove the pest killers. What do you do? There is both a permissive and mandatory disclosure here.

• Under 1.6, you could definitely disclose: there is a risk of substantial harm to other children!• 4.1: On some level, it’s a continuing danger: reckless endangerment, and you could be assisting the

criminal act if you don’t say anything.• Solution: You could give him a deadline by when to remove the rat poison units, and if not, you will

disclose/report it. • In real case, the prosecutor seized client’s records and found out the info. Harry did not destroy the

information and he handed over the records. A gvt agency went into the homes and removed all the units. Harry’s lawyer was able to make a plea bargain..

CHAPTER 4: ATTORNEY-CLIENT PRIVILEGE• CONFIDENTIALITY:

• Think about the ethical rules• Violation, subject to discipline• Much broader than attorney-client privilege• Structural—Relating to representation• Protect confidences at all cost• ALL information relating to the representation that a lawyer obtains (1.6a)• Subject to discipline under the rules

Page 14: Professional Responsibility Outline

• PRIVILEGE:• More narrow• Process-based rules/evidence law• Compelled disclosure—attorney/client asserts privilege• Communication between a lawyer and a client in which the client is seeking legal advice or other legal

services• Violation, results in motion practice/objection• The privilege is between the lawyer and the client when the client is seeking legal advice or other legal

services.• Motion to quash, objections to discovery• So some things are protected by confidentiality, but not privilege.• Conversations can be separated into non-privileged and privileged information.• Only the conversation is privileged, not the underlying facts• From client to lawyer and lawyer to client• Only LEGAL information is protected:• CHART ON 234

Ethical Duty to Protect Confidences

Attorney-Client Privilege

Source Ethical duty, rule 1.6 Common law evidence rule

Scope Information relating to the representation of a client (obtained from any source)

Narrower scope: confidential communication between a lawyer and a client for the purpose of obtaining legal advice

Method of Enforcement Professional discipline Quash subpoena or otherwise exclude the revelation from evidence.

←• ELEMENTS OF ATTORNEY-CLIENT PRIVILEGE:

• Communication: privilege covers face-to-face conversations, phone calls, memos, emails, faxes, IMs, etc.• Privileged Persons (communications with lawyer agents are also privileged- secretaries, paralegals,

investigators). Interpreters are covered too.• Communication in confidence: The client must reasonably believe that the communication is confidential• Communication for the purpose of seeking legal assistance: Communication is privileged only if the purpose

was obtaining legal advice (if they ask for business advice, like investment tips, it’s not covered)• If a lawyer doesn’t bill a client for the legal advice, it’s still privileged. You don’t need an exchange of

money• If part of a conversation is not about legal advice, that part is not covered under privilege• Privilege protects communication from client to lawyer, as well as lawyer to client.

• When communication is privileged:• The lawyer can’t be forced to testify

• Can’t testify over the client’s objective- the privilege belongs to the client and can only be waived by him• The client can’t be forced to testify

• The client may waive the privilege, even if the lawyer objects

Page 15: Professional Responsibility Outline

• Paper and electronic records of the privileged communication are also protected• Only the communication, NOT the underlying information is protected

• CLIENT IDENTITY• In general, identity of a client is not privileged. However, law is unsettled on whether identification is

protected if disclosure would incriminate the client in the very criminal activity for which the client sought advice in the first place.

• WAIVER• The attorney-client privilege can be waived expressly or by the lawyer is the client has authorized the waiver.• Waiver by inaction: if client is asked a question regarding privileged info in a deposition and the attorney fails

to object, and the client answers, it cannot be undone- privilege was waived• privilege can be waived if the client or lawyer reveals the info to a non privileged person.• Waiver requires a voluntary act by the client or an authorized agent of the client- so if lawyer on accident

reveals privileged info, there has not been waiver.• Problem 4-1: Murder for Hire (Page 247)

• Your client allegedly hired someone to kill his father. when you’re talking to him in jail you start to believe he actually killed his father. He agrees to admit guilt at first and then later changes his mind. They want to bring in inmate who overhead admission.

• Can you assert privilege and exclude the other inmate’s testimony? yes. • This was the only place they could meet so you could argue there was a reasonable expectation of privacy.

• Arguments that the attorney-client privilege applies:• Purpose was to deliver legal services• Client has reasonable expectation of privacy if this is the only place that jail provides to talk which

makes sense b/c• 6th amendment right to assistance to counsel

• Any communication was about legal strategy• Arguments that the attorney-client privilege does NOT apply:

• There was no expectation of privacy• The attorney-client privilege existed here.

• The Crime Fraud Exception:• There is no privilege if a client seeks assistance with a crime or fraud, even if the • No privilege for conversation if the client later uses the advice he received from the lawyer during the

conversation to commit a crime or fraud• criminal acts/fraud in the past are privileged• if a client consults a lawyer about a planned crime but doesn’t know that the conduct is criminal, the

communication is not privileged. As long as the client has the intention to perform an act that is criminal, even if they don’t know it’s criminal, it’s not privileged

• If a client asks about whether a certain act is permitted under the law, it’s probably privileged- as opposed to a request for advice to help commit a crime or avoid apprehension.

• Procedure for challenging a claim for privilege:• opposing lawyer must request documents based on guesses about what might exist or what it might

contain• the lawyer could argue for in camera inspection of the correspondence in which the judge reviews the

documents privately to decide if they are privileged, urging that the inspection might show intention to commit fraud. example of how important privilege claims can be: the tobacco litigation cases from the 90s.

• Problem 4-2: The fatal bus crash:• you represent estate of man who was injured in bus crash. his insurance won’t pay now because they say

the cause of death was not the crash.• You want to interview the insurance executives about the conversation they had with lawyers:• The insurance’s attorney will likely argue that Insurance- attorney client privilege applies.

• This was correspondence between them. • The purpose was to deliver legal services.

Page 16: Professional Responsibility Outline

• Any letters concerned legal strategy with respect to the claim. • There is no evidence that it has been shared with non-privileged persons. • And crime-fraud exception does not apply.

• HOWEVER, YOU SHOULD ARGUE THAT privilege has a narrower scope than confidentiality. If purpose was to make money, they are business records not privileged info. You can also argue that they are deliberately lying because they knew about medical records therefore, fraud exception does apply. Finally, argue for additional exception of wrong-doing so an intentional tort can be within the crime-fraud exception (new legal strategy).

• Real case: court looked at the documents, heard both sides, and concluded that the purpose of the crime-fraud exception would be effectuated by applying the exception to torts. So the court thought this was def. something that needed to be looked at, therefore no attorney-client privilege. Diamond v. Stratton 1982

• The Death of the client• Swidler & Berlin v. United States (The Dead Murderer Problem 4-3)

• Duty of confidentiality survives the death of your client. • Some people have suggested that we have exceptions (like for criminal prosecutions under federal law• What are the concerns? Civil liability, harm to family and friends, reputation...

• Problem 4-3: the dead murderer: Real case is Swidler: Your client is dying from cancer in prison, and he confesses to a murder that would let an innocent man go. Your client doesn’t disclose, then dies, and the other def goes on trial in a week and could get the death penalty- should you save the innocent def?

• 1.6: you could claim it fits under one of the exceptions b/c the other guy is in danger of substantial harm • What about under the rules of justice? You might want to find out more information before you disclose (look

at what kind of case the prosecution has against the “innocent” defendant)• 8.4 Misconduct: professional misconduct to engage in conduct that is prejudicial to the administration of

justice.• How long does the privilege last? no one really knows. We know that death does not end the privilege unless

he gives consent to disclose after his death.• Actual case: attorneys sought informal opinion from state ethics committee and the committee told them it

was permissible to disclose

WORK PRODUCT DOCTRINE Protects notes and other material that a lawyer prepares in anticipation of litigation from discovery in

pretrial civil proceeedings

• The scope of privilege for corporations:• Upjohn Co. v. United States: KNOW THIS FOR EXAM

• Attorney-Client Privilege for Corporations (FOR EXAM)• Control Group test- limits the privilege to communications from persons in the organization who have

authority to mold organizational policy or to take action in accordance with the lawyer’s advice. The reasoning behind this test is that we want to give people who have responsibility for company the privilege. However, the problem is determining who’s in the control group. The Supreme Court said that this test frustrates the privilege because low-level employees are not protected and therefore, won’t give or reveal any information. Renquist said we want a functional, practical test.  He then said that the privilege protects communications not underlying facts or certain groups of people. STILL, this test is used by several states because the subject matter test only articulates federal common law and states make their own evidentiary rule

• Definitional problems—who is in the control group?• Subject Matter Test- this is the test endorsed by the Supreme Court in the Upjohn case. In Upjohn, a

company was bribing countries to do business and the general counsel for the company decided to conduct an internal interview. This information was supposed to remain confidential. However, the

Page 17: Professional Responsibility Outline

company decided to voluntarily disclose some info to the SEC. Then the IRS issued a summons for the internal questionnaires promulgated to members of the company by the general counsel. Upjohn claimed that the questionnaires were privileged but the Sixth circuit applied the control group test. The Supreme Court rejected that test. As stated above, the Supreme Court said that this test frustrates the privilege because low-level employees are not protected and therefore, won’t give or reveal any information.

• More realistic view of how the organization operates• These low level employees are protected under this test

• Low level employees do work that the attorneys are interested in knowing• Corporation conducted internal confidential investigation• The sixth circuit limited notion of the privilege for the corporate counsel. The Supreme Court did not like this

and adopted the Upjohn Test: a broader test that they wanted a rule that reflected the way modern corporations function

• This case expanded the corporate attorney-client privilege in federal proceedings to communications between lawyers and low level employees who talk with a corporation’s lawyers to long as the lawyers are gathering information to help guide the company’s legal affairs.

• If they can determine it independently, it’s considered facts. If not, it’s communication (so you can ask the client about facts, but not about what legal discussions they had with their attorney).

• Client cannot be asked “what did you say or write to your attorney?”• The old approach was the “Control Group Test” and is still used in some states:

• “limits the privilege to communications from persons in the organization who have authority to mold organizational policy or to take action in accordance with the lawyer’s advice”- reason: we want to give people with authority privilege

• Problem is trying to figure out who fits into this group• “Upjohn Test”: The new “subject matter test” endorsed by Supreme court for cases based on federal law:

• “Extends the privilege to communications with any management or lower-echelon employee or agent so long as the communication relates tot he subject matter of the representation.”

• The McNulty memorandum: page 272****• If a legitimate need exists, prosecutors should seek the least intrusive waiver necessary to conduct a complete

and thorough investigation, and should follow a step-by-step approach to requesting information. Prosecutors should first request purely factual information, which may or may not be privileged, relating to the underlying misconduct.

• Under the McNulty Memorandum:• Category I: factual information• Category II: legal advice

• Whether or not to give over requests to waive attorney-client privilege with corporations: page 273:• Likelihood and degree to which the privileged information will benefit the government’s investigation• Whether the information sought can be obtained in a timely and complete fashion by using alternative

means that do not require waiver• The completeness of the voluntary disclosure already provided, and• The collateral consequences to a corporation of a waiver

• McNulty only applies to the Department of Justice.• Companies don’t have to abide by this- its just guidelines for the government for when government should

request waiver or attorney-client privilege or work product doctrine.• Fill in cases from chapter 4! • Problem 4-4: Worldwide Bribery: Read this problem!!!

• It’s category II info, so you shouldn’t just hand it over.• Three things you can do:

• Waive privilege and turn over records• Not waive the privilege (probably best answer: you shouldn’t give over confidential info every time you

have the slightest threat of prosecution.)• Middle ground: give some information, not all (this is the worst choice)

Page 18: Professional Responsibility Outline

•   i.      The pro is that if you turn it over you may be able to negotiate with the prosecutor. So, you may instruct the president to waive the privilege and turn over records. However, you may instruct him that he can give a little and hold some back. That way you are in a better bargaining position

•   ii.      The con is that if you turn them over, there is no limit to what the government can investigate or prosecute. Try to get something formal from the government first. Then turn them over.

•     iii.      You can also choose a middle ground. Just turn over cover and facts but not release any liability information or legal stuff.

←WORK PRODUCT DOCTRINE:• Must be in anticipation of litigation. If it’s a routine practice of the business/client to do something, and

then litigation arises, it probably won’t be privileged b/c it’s not in preparation for litigation.• It’s not absolute. A lot of the information you can obtain from other places.

• Showing of substantial need or undue hardship can overcome a claim of work product immunity when the allegedly protected material is a witness statement or other “ordinary” work product. “Ordinary” work product is that which is compiled by the lawyer, but does not contain the lawyer’s “mental impressions” (this is why it’s not absolute)

• The doctrine gives stronger protection to work product that reveals the lawyer’s thoughts, strategies, or mental impressions than it does to other forms of work product. But it doesn’t protect underlying facts.

CHAPTER 5: RELATIONSHIPS BETWEEN LAWYERS AND CLIENTS:• Elements of Attorney-Client Relationship• How Relationship is Established (See Togstad)• Agency (Lawyers as agent of Client Principal)

• Express and Implied Authority• Express- Actual

• Apparent Authority• What third party thinks

• Competence, Rule 1.1• Candor, Rule 3.3• Communication, Rule 1.4• Diligence, Rule 1.3

• Relationships between Lawyers and Clients• Choosing Clients:

• Can a lawyer accept work in an area of law where they have no expertise?• Well, rule 1.1 says competence requires the legal knowledge, skill, thoroughness and preparation

reasonably necessary for the representation.• However, if a lawyer can get up to speed, 1.1 does not prohibit the representation

• Can a lawyer bill a client for getting up to speed?• Yes. So long as it is not excessive.

• Example• A client hired a lawyer to represent her in a custody case. He had 48 years of practicing

security law but no experience doing custody. Still, he took the case. He billed her for more than $25,000 and pursued a bad theory of the case.

• Bar suspended him indefinitely for overbilling and not being competent.• How is the relationship formed?

• No formal agreement is necessary.• This means that you must be careful about giving casual advice.• A lawyer must also respond if they tell a person they will get back to them• If a person is waiting and does not get other counsel or if they miss a deadline because they are

waiting, a lawyer can be liable• What if a lawyer doesn’t want to take a case?

Page 19: Professional Responsibility Outline

• A lawyer is under no obligation to take any case but,• Rule 6.1 encourages lawyers to do 50 pro bono hours a year• Rule 6.2 says that a court may assign a case and you can only reject it for good cause• A lawyer cannot discriminate when they are choosing clients

• Legal Malpractice• Elements

• An attorney client relationship existed• That defendant acted negligently or in breach of contract• That such acts were the proximate cause of the plaintiff’s damages• That but for defendant’s conduct, plaintiff would’ve won case

3. HOW RELATIONSHIP IS ESTABLISHED (SEE TOGSTAD) • Togstad v. Vesely, Otto, Miller & Keefe:

• Togstad had an aneurism and a doctor surgically implanted a clamp in his neck to allow gradual closure of the artery where the aneurism was.

• Attorney, Miller, gave legal advice, did not qualify it. He said he would get back to her, and never did. He said she did not have a case. Never suggested she should talk to another attorney, and never told her he was not familiar in this area of law. He did not conduct the minimal amount of research. By the time she went to another attorney, the SOL had run. He gave legal advice and she relied on it.

• He did not do any research to assess the basis of the claim (clear evidence of malpractice)• The test for attorney-client legal relationship:

• Client sought and received legal advice• A reasonable person would have relied on it

• Rule: Four elements must be shown in this type of legal malpractice case:• Attorney-client relationship existed;• Defendant acted negligently or in breach of contract• Such acts were Proximate cause of plaintiff’s damages• But/for = successful (case within a case” see page 129)

• Apply it:• Don’t make statements about the law without adequate representation. You should explain the scope of

representation to make sure you’re covered (1.2)• Rule 6.2: Accepting Appointments:

• You should not accept a client if it’s likely to result in violation of rules, if it’s likely to result in unreasonable financial burden on the lawyer, or the client/cause is so repugnant to the lawyer as to be likely to impair the client-lawyer relationship or the lawyer’s ability to represent the client.

• Rule 6.1: Limitations on freedom to choose clients: • Lawyers have duty to provide legal assistance to people who are not able to pay for it.• Lawyers are encouraged to “aspire” to provide at least 50 hours/year or pro bono representation.• Lawyers may be assigned to represent indigent criminal defendants, even if they are not paid for it.• A lawyer may not discriminate on the basis of race, religion, nationality, sex, age, disability, or another

protected category in decisions about who to represent.4. LAWYERS’ RESPONSIBILITIES AS AGENTS: AGENCY LAW (3 AUTHORITIES)

a. EXPRESS: when the client says they want you to do somethingb. IMPLIED: an incident of doing the representation for the client (client’s not going to tell you to draft

documents and file them, etc.)c. APPARENT: only the acts or statements of client or another principle can justify reliance by a third party. If

you don’t have express authority, but you see your client doing something that gives you apparent authority. 5. LAWYERS’ DUTIES OF COMPETENCE, HONESTY, COMMUNICATION AND DILIGENCE A. COMPETENCE, RULE 1.1• 1.1: lawyer SHALL provide competent representation to a client. This requires the legal knowledge, skill,

thoroughness and preparation reasonably necessary for representation.• ***The MacCrate Report

• Identified ten things and four values that attorneys should have• See page 280-281

Page 20: Professional Responsibility Outline

• Ten Fundamental Lawyering Skills: • Problem solving• Legal analysis and reasoning• Legal research• Factual investigation• Oral and written communication• Counseling• Negotiation• Litigation and alternative dispute resolution procedures• Organization and management of legal work• Recognizing and resolving ethical dilemmas

• Four Fundamental Lawyering Values: • Commitment to values of attaining a level of competence in one’s own field of practice and

representing clients competently• Commitment to values of promoting justice, fairness and morality in one’s own daily practice• Participating in activities designed to improve the profession, assisting in training new lawyers,

trying to get rid of race biases in profession• Seeking out opportunities to increase their knowledge and improve skills

• Problem 5-1: The Washing Machine: • Hallmart, a retail appliance store• This problem is deceptively simple, which is where some of its complexity comes from.• Client doesn’t have money and offers a payment plan. We get through this quickly. The temptation and

inclination is to accept the store’s proposal. • Question: Whether representing this client competently requires further factual and legal research.• Options:

• Do nothing• Payment plan to settle• Get rid of this case

• Do more research...read the contract; under what terms did the client sign the contract? Were there any oral communications/agreements made to him? Are there warranties? This could lead you to counterclaim.

• Look at the company’s history for these claims and also see if they can discharged under state law• This does create a problem though b/c client may not be able to pay• You need to collect more information before negotiating• What should our objective be? • If you settled it, you wouldn’t be subject to discipline. You’re not being paid anyways, and the client is happy

(unless the contract says otherwise)• 1.2: Look at the scope of representation: you could limit it by agreement• No matter how much the case is worth, you should still do everything you can• 1.1: Competence

• Knowledge• Skill• Thoroughness• Preparation

• COMPETENCE IN CRIMINAL CASES: • 6th Amend requires that a criminal defendant be provided with a lawyer whose work meets at least the

minimum standard of being “effective”• Strickland v. Washington:

• Washington committed several crimes, including 3 murders. The state appointed Tunkey, an experienced criminal attorney to represent him in a capital case. Washington confessed to the murder and pled guilty against attorney’s advice. Washington had a sentencing hearing where “mitigating” evidence could be introduced but Tunkey did not offer character witnesses, etc. Tunkey knew that the judge liked individuals taking responsibility for their actions, so he did this as a strategy. Judge focused on

Page 21: Professional Responsibility Outline

aggravating circumstances. Washington submitted 14 affidavits from people who said they would have testified if asked to do.

• There’s still a case within a case: Look back at Togstad case. You have to prove that better representation would have made a difference. These are virtually impossible because you have such a high standard of what must be proven

• Rule 1.2 Scope of Representation• RULES:

• To prove that counsel’s assistance was so defective as to require reversal of conviction, you must prove two things:

• counsel’s performance was deficient. This requires showing that attorney made errors so serious that he was not functioning as the counsel guaranteed to def by the 6th amendment. (This is really difficult to show! they have to be totally ineffective)

• show that the deficient performance prejudiced the defense. This requires showing that the atty’s errors were so serious as to deprive the def of a fair trial, a trial whose result is reliable. So serious, that we have no faith that it was a fair trial.

• The judicial scrutiny of attorney’s performance must be highly deferential to the attorney• counsel has duty to make reasonable investigations or to make reasonable decisions that make

particular investigations unnecessary.• an error by counsel does not warrant setting aside judgment if the errors has no effect on judgment.• It’s not enough that the def show the errors had some conceivable effect on the outcome of the

proceeding (b/c almost every act would meet this)• APPLICATION: The conduct of the counsel here is not unreasonable. Washington suffered insufficient

prejudice to warrant setting aside his death sentence. Tunkey’s actions were strategic decisions.• DISSENT strongly argues that counsel is afforded too wide of a latitude and that the requirement of

acting like a “reasonably competent attorney” is too vague, it’s “to tell them almost nothing”. It’s unreasonable for counsel to fail to investigate.

• Is this reasonable? Was the attorney reasonable? Powell thinks not. Here, unlike in the Washing Machine problem, one’s liberty is at stake and perhaps you should do more anyways!

• Defendant was executed in 1984, 2 months after court’s decision. There was a compelling dissent that said lawyer should have brought up everything.

• After this case, for a long time, only 4 percent of criminal def’s won on incompetency charges against their lawyers.

• Notes• Attorney is the agent of the client and the attorney has the discretion to use the proper means to

hold up to the client’s objectives• Is this standard to deferential? Not really, bc it was the client’s fault that he made the decisions

that he made and he cannot blame the attorney for his choices• So deficient that its almost like performing without an attorney—attorney was not being used as a

counsel should be used like the 6th Amendment states• Does this case give a definition of Competence?• To a certain extent, it is all in our hands as attorneys• Standard is so deferential that it is almost impossible to lose

B. CANDOR 3.3 AND COMMUNICATION 1.4• Is it ever okay to lie?

• Questions to ask• is the subject trivial or private?• is anyone harmed by the lie?• is the purpose of lying to protect someone?• does the person lied to have a right to know?• is there a reason to tell the lie, can the problem be solved without lying• if you lie, will it require follow-up lies to cover it up?

• Problem 5-2: Lying to Clients:• lowballing

Page 22: Professional Responsibility Outline

• i never reviewed this document• who did the work?• covering up mistakes• blaming others for mistakes

• Rule 4.1: Truthfulness with persons other than clients • Rule 3.1: Honesty toward tribunal • CMT 7 to Rule 1.4: lawyers may withhold a psychiatric diagnosis of a client when the psychiatrist indicates

that disclosure would harm the client. BUT, the comment does not address whether the atty can lie if directly asked by client about it.

• Generally, lawyers should be honest with clients and others..(although The rule does not explicitly require lawyers to be honest with their clients)

• Rule 8.4(c) prohibits attorneys from conduct involving dishonesty, fraud, deceit or misrepresentation• YOu have to have knowledge that you’re lying in order for it to be forbidden.• You can lie in negotiations • definitions of fraud vary from state to state. If a lie is not fraud it still may be deceit or misrepresentation.• CMT 1 to rule 1.4 also requires “reasonable communication b/w lawyer and client as necessary for the

client to effectively participate in representation.” they can’t do this if they’re lying!• Lawyers must inform clients of important developments in their case• Rule 1.13: an organization as a client: org’s don’t lie, people do.

• RULE 1.4: COMMUNICATION • Lawyer must INFORM CLIENT promptly of any decision or circumstances if informed consent is required

for anything• Reasonably consult with the client about the means by which the client’s objectives are to be achieved• Keep client reasonably informed about status of their matter• Promptly comply with reasonable requests for information• Consult with client about relevant limitation on lawyer’s conduct when the lawyer knows that the client

expects assistance not permitted by the Rules• Explain matters to the extent reasonably necessary to permit the client to make informed decisions• Have to communicate something that may impact representation• Communicate about the means that will be taken to reach the client’s objectives• Seen as a collaborative relationship, but the attorney must make decisions to control the matter

• Civil liability for dishonesty to clients: • Can sue the lawyer in tort for fraud or for breach of lawyer’s fiduciary duty

• RULE 2.1: CANDOR: • Lawyer shall exercise independent professional judgment and render candid advice. In rendering advice, a

lawyer may refer not only to law but to other considerations such as moral, economic, social and political factors, that may be relevant to the client’s situation.

• CMT 1: A lawyer should not be deterred from giving candid advice by the prospect that the advice will be unpalatable to the client.

• Rarely any complaints filed under this rule (mainly because it occurs in private)

• PROBLEM 5-3, Torture: • Context where all parties considered clients want a forward looking approach, to make torture

permissible, or at the very least, makes it a defense that the counsel told them they could do it. • One important consideration is that you want to protect your client from future litigation. We must give

bad news when necessary.• You can give them the narrow, forward-looking memo they want, but still be candid and warn against

pitfalls.

Page 23: Professional Responsibility Outline

• 2.1: Advisor: lawyer shall exercise independent professional judgment and render candid advice. In rendering advice, a lawyer may refer not only to law but also to other considerations such as moral, economic, social and political factors that may be relevant to the client’s situation.

• Comment 1: lawyer should not be deterred from giving candid advice by the prospect that the advice will be unpalatable to the client.

• It’s very rare for a 2.1 to be grounds for violation charge. no one has ever been disciplined under 2.1• 5.2b: may be a reasonable resolution of an arguable question• The lawyers should edit the memo to reveal doubts about the Medicare standard and Youngstown Sheet

& Tube Co. v. Sawyer• You’re probably violating 3.3 if you don’t talk about Youngstown Steel case

C. DILIGENCE• Rule 1.3: A lawyer shall act with reasonable diligence and promptness in representing a client.• Common violations: failing to return calls or not filing papers with court on time.• CMT 1: lawyer should pursue matter for client despite personal opposition or inconvenience and take

“whatever measures are required” to vindicate client’s cause• CMT 3: perhaps no professional shortcoming is more widely resented than procrastination• Canons used to say you were required to act with “zealous advocacy” but this was problematic because it

did not mean you should act hostile towards opposing counsel

D. WHO CALLS THE SHOTS?• Rule 1.2: Scope of representation and allocation of authority b/w lawyer and client• Jones v. Barnes:

• We preserve the lawyer’s autonomy• There is no 6th amendment right for a client to demand which issues to raise on appeal• the rule is based on underlying theme of collaborative decisions b/w lawyer and client.

• Problem 5-4: The Package Bomber: (are we lying to our client) • You represent a package bomber. On appeal, you think your best argument to save him from the death

penalty is to claim he has paranoid schizophrenia. He will not consent to psychiatric exam though and does not want you to argue this. Instead, he wants you only to pursue the procedural arguments (that the search and seizure was illegal).

• Options:• Go with the client’s wishes and try to persuade jury that his bombings were necessary to warn and

protect society from a greater disaster. terrible idea.• Despite client’s objection, ask the judge to order a psychiatric evaluation. you may not be able to force

him to submit.• A possible compromise: trick the client into undergoing examination and then use it. this may be the only

real option. • In the real case, the attorneys kind of tricked the client and used the evidence. the client was upset and sued

the attorneys asking for post-judgment relief, but the court rejected the argument. He argued that the evidence undermined his defense. court said the attorneys were fulfilling their duties.

• Relevant Rules:• 1.2: Scope of Representation (Candor)

• Important because it tries to divide up things in a good manner• (a) a lawyer shall abide by a client’s decisions concerning the objectives of representation and

shall consult with the client as to the means by which they are to be pursued• 1.4: communication• 1.14: Diminished Capacity rule- “Maintain a normal client relationship”

• (a) … Capacity to make adequately considered decisions in connection with a representation is diminished… the lawyer shall (mandatory language), as far as reasonably possible, maintain a normal client-lawyer relationship with the client

• What is diminished capacity? Comment 1• Comment 2

Page 24: Professional Responsibility Outline

• Comment 5- protective action• Comment 6- factors for diminished capacity

• 1.6 might be impliedly authorized to reveal info.• 1.1• 1.3• 8.4

• We want to maintain client autonomy and not be overly paternalistic. But who should call the shots?• You have to balance Client Autonomy and Paternalism.• What happens when the lawyer and the client disagree about the means to achieve the client’s objective?• Is there a constitutional duty to raise every issue in a client’s frivolous claim

• If not, why not?

CLIENTS WITH DIMINISHED CAPACITY:• Rule 1.14• Guardianship is only appropriate in extreme cases• PROBLEM 5-5: VINYL WINDOWS:

• You go to client’s home because she can’t come to the office. She is an elderly widow, the living room is cluttered. She tells you that two salesmen came to her house to work on her windows, and talked her into signing papers, and she went to the bank and withdrew money to give them. When they started working on the windows, she would go to the bank and withdraw payments. When they asked for the final 900, she didn’t want to pay it b/c the windows were leaky. Now they’re suing her and she can’t recall what she paid, when, etc.

• We would try to pursuit what she wants, but try to explore everything. • Real case: facts were a lot harsher. McCabe was found staring at an open fridge in her home. The lawyers

were worried about future problems with salesmen • 1.14 try to have a normal client relationship. here you can’t so try to get a guardian ad litem- but this is

probably not necessary- it’s too early at this point. would violate client autonomy.• talk to neighbors...?• balance client autonomy with paternalism• do what you can to get her to decide

• PROBLEM 5-6:• You are assigned to represent client- the state is going to ask that the client be admitted for mental care. You

discover a procedural error in the commitment petition and go to tell him about it. When you visit him, he is angry, aggressive, and very scary. should you still tell him that you have a way to keep him from being committed?

• Probably try to get the client released. His actions don’t dissolve the relationship and you’re not required to withdraw.

• READ Rule 1.16- Declining or Terminating Representation

JUVENILES:• children are owed the same duties of undivided loyalty, confidentiality, and competent representation to the

child as is due an adult client.• PROBLEM 5-7:

• Child does not really know what she wants. She’s staying with a foster parent now that has a lot of experience, but not with special needs children. Grace wants to return to her mother who abused her (child does not really remember the abuse).

• Can you respect client autonomy while doing what’s in their best interest?• you DO NOT have to follow Grace’s objectives b/c she may not be able to make an informed decision• you need to determine if she is capable if making decisions under 1.14 and if so, follow her wishes.• some law reviews also say that children need and often expect that an adult will make important

decisions for them.• being assigned as her guardian would be going too far

Page 25: Professional Responsibility Outline

• maybe just act as reporter to court• the best option is probably family reunification• 1.4 comment 6

TERMINATING LAWYER-CLIENT RELATIONSHIPS:• 1.16:Declining or terminating representation:

• 1.16D: Upon termination, lawyer should take steps to the extent reasonably practicable to protect a client’s interests, such as giving notice to client, surrendering papers and property to which the client is entitled, refunding advance payments or payments not used, etc.

• PROBLEM 5-8: Candid Notes: After you stop representing client, they request their entire file. Their file has a lot of candid notes written on it about impressions of the client, etc. Do you hand it all over?

• the client does not “reasonably need” the information in this internal memorandum so you probably would not give it to them. tell the client that you think they want all court documents, etc., but that you are excluding internal documents that you withhold from all clients.

• These are internal documents intended for internal review so the client does not need them and lawyer can refuse to hand them over.

• Don’t redact because it would raise suspicion.• GROUNDS FOR TERMINATION:

• A client can fire you at any time and for any reason. • MUST WITHDRAW:

• you must withdraw if you’re fired.• you must withdraw if you are ill or incapacitated, (physical or mental health)• or you know your representation would be materially impaired• if continued representation would violate ethics rules or the law

• PERMISSIVE WITHDRAW:• no harm to the client• Even if material harm will occur, you still may be able to if:

• lawyer reasonably believes client is acting criminally or fraudulent (this is different from KNOWING they are, in which case withdraw is required)

• past use of service for crime or fraud (even if it’s not currently be used for these purposes) rule 1.16b3• client’s views are repugnant or imprudent (their views are very terrible)• client fails to meet obligations (like paying; first you must warn the client)• unreasonable financial burden• client unreasonably difficult to work with (continual failure to appear, missing appointments repeatedly)• if you have appeared as the attorney in the case, you can’t just withdraw and leave.

• Even if the lawyer has good cause to withdraw, the court may order the lawyer to continue in which case they must.

• DUTIES UPON TERMINATION:• Fee refund of any fees paid but not yet earned• Client’s papers and property• Client continued to have an obligation to pay fees already earned by the lawyer 1.16d

CHAPTER 6: CONCURRENT CONFLICTS OF INTEREST: GENERAL PRINCIPLES:CHAPTER 6: CONFLICT• Things that impact and can undermine the attorney-client relationship.• In the 1940’s, 50s, and 60s, it was not uncommon for attorneys to represent opposing clients.• Conflict is factually explicit, there are language barriers because of technical lingo• Identify problems and try to resolve:

• Withdraw• Not accept representation• Is the conflict consentable?• Is screening allowed?

Page 26: Professional Responsibility Outline

• RULE 1.7: Conflict of Interests: Current Clients:• A lawyer SHALL NOT (mandatory) represent a client if it involves a concurrent conflict of interest:• 1.7(a) A concurrent conflict of interest exists if:

• The representation will be directly adverse to another client (see comment 6)• There is significant risk that representation will be materially limited by lawyer’s responsibilities to

another client, a former client, or a third person or by a personal interest of the lawyer (see comment 8)• 1.7(b) In spite of a concurrent conflict of interest, a lawyer MAY represent a client if:

• Lawyer reasonably believes he can provide adequate representation to each affected client• HOW DO YOU KNOW IF YOU CAN PROVIDE ADEQUATE REPRESENTATION? See

checklist of questions on page 364.• The representation is not prohibited by law• The representation does not involve the assertion of a claim by one client against client represented by

the lawyer in the same litigation or other proceeding before a tribunal, and• Each affected client gives informed consent, in writing.

• If you don’t get the informed consent, you can’t represent/continue to represent• Comment 2 tells you the questions to ask!

• If you want to know whether or not you have a conflict, you should ask:• Clearly identify the client(s) and interest(s)• Determine whether a conflict of interest exists

• Is it directly adverse? Or materially limited?• Conflict exists, but is it consentable?• Consult with the clients affected and obtain their informed consent

• Get informed consent in writing• No matter how big the firm is, and where its offices are, we treat the entity as ONE lawyer. Rule 1.10!!

• PROBLEM 6-1: INJURED PASSENGERS: TAXI: 403-404:• No direct adversity here• two women in a taxi when it crashes. the women do not know each other. one just has whiplash, and the other

has fractures, more severe injuries. If you were to represent both, you would charge based on contingency fees.

• we don’t have any DIRECT adversity, so then we go to material limitation (like insurance policy caps).• Is this a non-consentable case? no. As long as everything is open and clients agree, it’s okay.

• What to disclose:• Explain loyalty, confidentiality and attorney -client privilege• Advantage/risks of proceeding together• How confidential info will be handled/shared among clients• Come up with an alternate plan incase you can no longer represent both (like one will leave and seek other

counsel). • You should memorialize their agreement to your representation.

• Becomes much more complicated when settlement arises. What happens when we get a settlement that’s different for the two clients?

• You can share confidences. • You need a contingency plan in case conflict arises. A plan that allows you as an attorney a way out of the

joint representation (Comments 30 and 31) when interests of the parties diverge

CONFLICTS B/W CURRENT CLIENTS IN CIVIL LITIGATION:• You represent Fred in his divorce. A new client, Mona, comes to you about suing Fred for car accident.

This is DIRECTLY ADVERSE.• If you were to represent Mona, you would know about Fred’s assets and values

• PROBLEM 6-2: I THOUGHT YOU WERE MY LAWYER:• One of your firm’s clients was hit by a bus and she’s suing the bus company.

Page 27: Professional Responsibility Outline

• Firm also agreed to represent a man in a divorce, who now you know is the husband of Doris (bus accident client)- you didn’t realize relationship b/c their last names are different.

• The firm should at least withdraw from representing the husband• Analyzing under RULE 1.7:

• Try to get informed consent (see what to disclose above)• If you can’t get informed consent, you will need to get rid of one or both clients (if both have

provided confidential information adverse to the other).• In the real case, the firm was fired from the personal injury matter when they found out firm was

representing spouse. Firm sued for fees for work already performed. Court held firm should have received informed consent, but the firm WAS entitled to some fees.

CROSS-EXAMINING A CURRENT CLIENT:• You represent a criminal defendant (1), and another person (2) who is charged in unrelated crime. 2 is gong

to testify against 1 which means you will have to cross-examine 2. REPRESENTATION OF CO-PLAINTIFFS OR CO-DEFENDANTS IN CIVIL LITIGATION:• Several states have held that a driver and passenger have sufficiently divergent interests that they may not

be represented by a single atty. The Restatement however takes position that this type of conflict may be waivable by the client after full disclosure.

• PROBLEM 6-3: INURED PASSENGER 2:• You represent 1 and 2, injured passengers (you have already gone through all the procedures in 1.7b to make

sure you can represent both). One’s injuries are worse than the other-The insurance company in settlement negotiations hints that he might offer 350,000 to settle more injured case, if you’ll accept only 50,000 for the less injured.

• What to do? You need to properly • What if you find out one of the clients were drunk, which contributed to the accident? you would need to

consult with them b/c they’re adverse now.• It’s a more serious conflict and if it’s a cross-claim, it’s not consentable.

• How can your representation of one client be “materially limited” by representing the other?• 1.7 QUESTIONS to always ask:

• What are the client’s objectives?• How can we achieve them?

• we know there is a “material limitation”. Is this situation consentable? • This situation would NOT be consentable if both clients wanted the maximum payoff.

• The economics would say this was consentable. Because of the “material limitation”, you would need to Consult with your clients and get their consent.

• COMMENT 6 to 1.7: simultaneous representation for clients who are only economically adverse, does not ordinarily constitute a conflict of interest.

• Rule 1.9 – duties owed to a former client (if Jill was dropped)• Would be advisable to withdraw from both clients

• PROBLEM 6-4: PRISONER’S DILEMMA:• There’s two class actions, one for women’s prison, and one for facilities for mental retardation. The lawyers

for both sides are part of same legal assistance program, different offices (doesn’t matter, same entity)• There are two different interests:

• Pellegrino: Women’s prison is not adequate, no exercise areas, libraries, etc.• O’Conner case: Mentally disabled facility is overcrowded.

• State offers to continue reducing numbers of mental facility, and move women into the mental facility which had better resources, build security there.

• Is there material limitation? Yes, seems to be a material limitation here• There’s a significant risk that services to one are limited by services to other.• You can try to talk to everyone.• Look at 1.10(a)

Page 28: Professional Responsibility Outline

• While lawyers are associated in a firm, none of them shall knowingly represent a client when any one of them practicing alone would be prohibited from doing so by Rules 1.7 or 1.9

• Real case: the court looked at the limited representation, etc., and one issue that came up: did the state make the offer in order to make a conflict to disqualify the legal services firm? Court said no, said the state made good faith effort to negotiate. There was clearly material limitation to the representation. Ultimately, court ruled that there should have been disqualification- eventually, the women’s prison was built, but only after years of appeal.

TAKING INCONSISTENT LEGAL POSITIONS IN LITIGATION:• “POSITIONAL CONFLICT”: inconsistent arguments on a legal issue in different courts at different times.

(Sometimes, even though you are arguing contradicting arguments which could create precedent affecting your other client, positional conflict is okay. you have to determine whether or not it’s okay.)

• Factors to consider whether there’s a positional conflict that creates a serious problem:• whether issue is before trial or app. court• whether issue is substantive or procedural• the temporal relationship between the matters• the practical significance of the issue to the immediate and lon-run interests of the clients involved• clients’ reasonable expectations in retaining the lawyer.• Set up an agreement for the scope of representation with your client under rule 1.2 letting them know that you

can represent them up to a certain point when a particular limitation arises• PROBLEM 6-5 TOP GUN:

• Large law firm was representing city in lawsuit against gun manufacturers b/c of high gun crime rate. • Other client, a pharmaceutical company, believes that your representation against gun companies could create

dangerous precedent for their liability of abused drugs. They threaten to no longer use your firm if you continue in gun case.

• We can’t look at obtaining consent since they are already upset• You do NOT have to withdraw-

• Argue there is no real risk/real conflict: • This is trial court, not appellate court so there is limited precedent value• Global’s concerns are hypothetical, not about present tense litigation• There is no real proximity between these issues. you’ve already been working for 2 years, and it’s 6

months before trial so the city is relying on you and you’ve already invested much time and money. • The gun manufacturer that called global may be playing on his own fears of liability- you may want to

thoroughly explain the low risk• Adverse publicity could result if you withdraw from city, and you might demoralize some of your

attorneys.• Maybe it’s a good business decision to withdraw. The noble position is not to withdraw: there’s no real

material limitation and the city really needs this representation.• Look at the factors on page 415 to see if you have a positional conflict that rises to unacceptable level.

• Issue is before a trial or appellate court?• Issue is substantive or procedural?• The temporal relationship between the matters• The client’s reasonable expectations in retaining the lawyer

• Here, there is no materially limited conflict here • It is at trial court—purely persuasive authority--- level of the court is so low so no concern for

prejudicial value• Actual cause:

• Weil, Gothsal, Manges: they withdrew from the gun case

CONFLICTS INVOLVING PROSPECTIVE CLIENTS:• There are obligations even to someone who isn’t really a client.• RULE 1.18:

Page 29: Professional Responsibility Outline

• In consultation, no matter how long it is, you can’t reveal info learned during that consultation, except for exceptions under 1.9.

• 1.18c: • See Comment 3!!!• The amount of time spent with the prospective client does not matter- it’s an objective assessment• When the attorney receives the disqualifying information, they may still be able to represent client

according to exceptions under 1.18d.• Comment 5 could also be a way around (exception) to 1.18. It allows lawyer to tell prospective client that

the attorney will not be barred from representing another client in the matter so the information disclosed may be used.

• refer back to TOGSTAD case

• PROBLEM 6-6: THE SECRET AFFAIR:• You have info from prospective client, Maria, that would hurt her. your firm already represents her husband

in divorce. Maria is a prospective client under 1.18a• what about 1.18d2?• the website form she submitted the info on had a small disclaimer• Is the firm prohibited under 1.18 from using/revealing info in the consultation?• REAL CASE: Court said the website really prevented the formation of a explicit contract with Maria.

Second, the disclaimer was sufficient to prevent formation of implied contract. The disclaimer of confidentiality was “unclear,” and the court said the firm “may have to stop representing” Nicholas and there was a duty of confidentiality.

• The information cannot be used.

Chapter 6 Recap:Ask yourself:• what is the source of the conflict?• does it meet the particular rule’s threshold requirements?• is the conflict one that imputes to the entire law organization?• if so, can the affected lawyer be effectively screened or isolated?• Is the conflict of a type that allows client waiver?• If so, what has to occur for the waiver to be effective?

• Conflicts:• Material limitation: as a general rule, conflict of interest exists when a lawyer’s representation will be

materially limited by the lawyer’s personal interests• Waiver: Waiver of general conflict is possible when the lawyer reasonably believes that she will be able to

provide competent and diligent representation to the client and the client gives informed consent, confirmed in writing.

• Business transactions: 1.8a• It’s required that the client’s consent be in writing, that

• the client be ‘advised in writing of the desirability of seeking and is given a reasonable opportunity to seek the advice of independent counsel,”

• that the transaction be objectively reasonable,• that the transaction be in writing and in terms that can be understood by the client and• the client gives informed consent in writing

• Lawyers are prohibited from negotiating for literary rights or media rights based on the subject of representation until the conclusion of representation

• A lawyer is prohibited from drafting documents that make substantial gifts to the lawyer or the lawyer’s close relatives, unless the donee is related to the donor.

← CHAPTER 7: CONCURRENT CONFLICTS IN PARTICULAR PRACTICE SETTINGS (RULE 1.7)

Page 30: Professional Responsibility Outline

• RULE 1.7: Lawyer shall not represent a client if the representation involves a concurrent conflict if interest.• Rule 1.8 :

• Conflict of interest with current client: Lawyer shall not enter into a business transaction with a client or knowingly acquire an ownership, possessory, security, or other pecuniary interest adverse to a client unless:

• full disclosure of transaction and terms in writing, and it’s fair and reasonable to the client;• client is advised in writing of the seeking and is given a reasonable opportunity to seek the advice of

independent legal counsel on the transaction; AND• the client gives informed consent in writing (signed by client) to the essential terms or the transaction and

the lawyer’s role in the transaction (including whether the attorney is representing the client in the transaction)

• lawyer shall now use the info relating to representation of client to the disadvantage of the client unless the client gives informed consent, except as permitted by rules

• lawyer shall not solicit any substantial gift from client, including testamentary gifts• prior to conclusion of representation of a client, lawyers shall not make or negotiate an agreement giving the

lawyer literary or media rights to a portrayal/account based on representation• lawyer shall not provide financial assistance to a client in connection with pending or contemplated litigation:

there are two exceptions. see rule 1.8(e)(1-2)• can’t accept compensation for representing a client from one other than the client unless

• client gives informed consent• there is no interference with lawyer’s independence of prof. judgment or the C-L relationship

• you cannot make aggregate settlement of claims when you rep more than one client unless you have informed consent

• see rest of rule...

REPRESENTING ORGANIZATIONS:• RULE 1.13: Organization as a client

• (a) Organizational Entity• (b) Reporting up• (c) Exception, see (d) Reporting out: clearly a violation of law• (d) Investigation

• PROBLEM 7-1: A Motion to Disqualify• KJR (Harris Kohn) v. Baby Care (Wholly-owned subsidiary of Melton)- represented by Alston &

Associates who move to disqualify Harris Kohn because HK also represents Melton• Should the motion to disqualify be granted? – look at rule 1.13• Arguments for motion for disqualification

• Baby Care= Melton bc shared legal advice, payroll, accounting, other services• Direct adversity (directly adverse to Melton) (Rule 1.7a1 and 1.7a2)• Disloyal to Melton• Consent?

• Arguments against motion for disqualification• Separate entity from Melton• Not part of the contract that gives rise to the current dispute• No Confidences from Baby Care• Financial impact? (Only a small impact on a separate entity)• KJR should be able to choose its own counsel• Burdensome on law firm• Unrelated?

• Court said the law firm Harris Kohn should be disqualified

Page 31: Professional Responsibility Outline

Problem 7-2 My Client’s Subsidiary• Dori Hathaway (represented by Shelton & Cadenas) v. Pearl Bus Co. (another firm represents this company, but it is owned by Transport, Inc. who is a client of Shelton & Cadenas)

• Have to withdraw from representing Hathaway if Pearl is considered a client because owned by Transport, Inc.

• Definitely cant do anything directly adverse (such as suing ourselves—suing Pearl is suing ourselves)

Representing Co-defendants• Representing one defendant can be adverse to the other client• RST and other commentary says that it’s a bad idea (but it can be done)• Convictions can be reversed by improperly requiring joint representation by a judge when the lawyer had timely objected saying that there is a conflict

• Courts will overturn conviction and reverses • If no one objects to joint representation of defendants, a resulting conviction may nevertheless be overturned if the conflict significantly affected the representation.• If a defendant who was jointly represented is convicted, and there was no objection at the time, to overturn the conviction on Sixth Amendment grounds, the defendant must show that there was a conflict of interest that actually affected the adequacy of his representation• A court may disqualify a lawyer on the basis of an actual or potential conflict even if the defendant prefers to waive the client

• PROBLEM 7-3: POLICE BRUTALITY:• Alston was the victim of police brutality- assaulted by multiple officers. • Tom Babbage is a police officer and pled guilty to charges • Two other officers, Chip Stone and Bob Morton, are co-defendants. Unsure of their guilt- The PBA has asked

you to represent these two.• PBA: provides legal fees and lawyers to represent police officers. This is an institutional entity that is paying,

and has broad ranging interests.• There is another officer who is also charged, and is a board member of the PBA- he has separate counsel.• ISSUES:

• 3rd party is wanting to pay legal fees: can you maintain your independence and provide your best representation or are they going to want to limit fees?

• (Looking at possible conflicts) In this type of situation look at Rule 1.8f• Shall not accept payment from a third party for legal services UNLESS your client gives informed

consent, there is no interference with the lawyer’s independence of professional judgment or with the client-lawyer relationship, and info relating to representation of a client is protected as required by Rule 1.6 (basically, confidences are protected)

• In this situation it’s NON-consentable• Rule 1.7 Conflict of Interest of current clients (Former clients and potential clients)• Conflicts :

• Two officers as your clients—either one could have done it—one defendant will point to the other and will drive a wedge between our co-defendant clients (assigning blame)

• Cross examination (Can we aggressively cross-examine without hurting the other client)• Inculpatory evidence/confidences• Rule 1.10- your law firm is one attorney basically

• Are the conflicts consentable under Rule 1.7a • Court ruled that the client’s waiver of conflicts of interest is not waivable because the attorney’s motive in

representation was based on the $10 million contract

• PROBLEM 7-4: POLICE BRUTALITY SCENE 2:• what if everyone consents,

• they give informed consent• there’s a common interest in defense of any officer accused of wrong doing

Page 32: Professional Responsibility Outline

• then can you accept the 10 million dollar contract?• Could you competently and effectively represent the clients (co-defendants)

• PROBLEM 7-5: SCENE 3• What if one of the clients refuses to waive any conflicts? • Can you fire him to keep the more lucrative client?• Most courts really disapprove of selling your client out like this (See “hot potato doctrine”)• What if you find out that one of the officers was in the bathroom at the time and pushed the victim into the

bathroom. • Is this conflict consentable? You would need to get consent, but you can’t revel the info without revealing

important info.• Real Case: testimony of the other officers was NOT introduced. Attorney relied on theory that Officer

Babbage acted alone. He pled guilty. Then Stone was convicted and appealed claiming “ineffective assistance of counsel” b/c of your conflicting interests of PBA and its board member Gutman, and that despite his consent to your representation, the judge should have disqualified you to prevent the violation of his 6th amendment right to effective counsel. (So he waived, but was it consentable??). Court found conflict was non-waivable- the biggest conflict was attorney’s interest in keeping $10 million agreement with PBA. The attorney’s self-interest was too strong

RERESENTING FAMILY MEMBERS : • Representing both spouses in a divorce:

• Florida Bar Opinion 95-4 (1997): What if you’re representing both husband and wife in preparing will, and after you write it, the Husband tells you he’s changed his will and wants to leave property to mistress.

• You must withdraw. You are bound to protect Husband’s confidentiality and at the same time, remain loyal to Wife.

• There would be undivided loyalty to both parties.

• Problem 7-6 : The McCarthy’s: You represent Husband and Wife. When clerk files their info away, she misspells their last name. Later, another woman hires firm to represent her in a paternity issue- she is saying the Husband is the father of her child. Firm doesn’t realize they represent Husband b/c of the typo/misspelled last name.

• There is a direct conflict of interest here.• K&D Represent Hugh (Husband) and Jolene (wife)—will/estate• K&D (Maureen Carr/Gus Kenney) V. Hugh (Paternity/child support)• 1.10 Imputation• 1.7: Conflict of Interests: Current Clients

• Consent is NOT possible here. 1.7 Current Clients 1.9 Former Clients

Directly adverse Same or substantially related matterOr materially limited And materially adverse to the interest of former client

Possibility of informed consent Possibility of informed consent(more restrictive) (less restrictive)

See Comment 3• Can you just drop one client and continue to represent them? You have to analyze former clients under

Rue 1.9• The lawyer’s duty of confidence (Rule 1.6) is greater than the duty to communicate to a client info that is

relevant to representation• The best plan is to withdraw from all three parties. You have a continuing duty to advise Wife in

estate planningRule 1.8 f: a lawyer shall not accept compensation for representing a client from one other than the client unless:

1. The client gives informed consent

Page 33: Professional Responsibility Outline

2. There is no interference with the lawyer’s independence of professional judgment or with the client-lawyer relationship; and

3. Information relating to representation of a client is protected as required by Rule 1.6

REPRESENTING INSURANCE COMPANIES AND INSURED PERSONS• The lawyer is being paid by one client (the insurer) to represent both itself and another client (the insured).

The insurance defense lawyer’s dilemma is fundamentally about money b/c the lawyer is being paid by the insurer.

• If there is a conflict b/w insurer and insured, the lawyer should act in the best interests of the insured, except that the lawyer may not assist client fraud. If they insured is also the lawyer’s client, the lawyer should try to act in the best interests of both clients. If not possible, they must withdraw from both. if there is a conflict, the insurer will pay the cost of hiring a separate lawyer for the insured. Most jurisdictions have held that if the insurer contests coverage, the insurer has to pay for a lawyer selected by the insured unless there is a prior agreement otherwise.

• Problem 7-7 Two Masters:• Insurance attorney was told by insured not to settle a claim w/o her permission. When he called the adjuster,

they said they attorney did not need to call the insured to settle. “I’ll deal with her- you settle the case”...• The attorney did not settle and he caught a bunch of hell for it.• The insured probably did not want to settle w/o her approval because even though insurance co would pay for

it, her premiums would change.• The firm was upset because the insurer, the insurance company, was the one paying bills! • The third party can’t decide the settlement; your client has that right.• Why might the partners of the firm have been unhappy that the attorney wanted to consult with the insured

person before agreeing to a settlement?• The firms treat the insurance company as the client

REPRESENTING PLAINTIFFS IN CLASS ACTIONS• There can be internal conflicts in a class action• Class actions are OUTSIDE the traditional rules of conflicts. We tolerate a level of conflict in these cases

b/c we have hundreds, if not thousands of clients, and there’s going to be some internal conflict.• The overarching theme is that we try to get “broad societal public relief” (many of these ppl do not have

access to representation). • On some level, conflict is a natural part of class actions.• One thing that we look at• Exception to undivided loyalty to clients, removing conflicts

• This is hard to do in a class action• We are tolerating some conflicts in the name of access to the system, resolving conflicts• We are encouraging some intra-conflict within the class action to resolve a larger conflict

REPRESENTING PARTIES TO AGGREGATE SETTLEMENTS OF INDIVIDUAL CASES• Difficulty is getting agreement among people in the aggregate settlement• Rule 1.8g• Cmt 13 to 1.8 tries to address this....• You may not practically be able to meet requirements when working wit hundreds of clients..

CHAPTER 8:CONFLICTS INVOLVING FORMER CLIENTS

Difference between Concurrent conflicts (1.7 and 1.8) and successive conflicts (1.9) Concurrent Conflict- forward representation, looking at what can happen to a current client

o Two current clientso One current and a potential client

Successive Conflict- looking backward at what could happen to a former client

Page 34: Professional Responsibility Outline

o A current client and one former client that we representedo RULE 1.9- Looking to see if there is a conflict that is the same or substantially related matter and

the interests are materially adverse to the interests of the former client unless former client gives informed consent, confirmed in writing

Break down of RULE 1.9 (a) A lawyer cannot represent a new client if that client has the same or substantially related matter and is

materially adverse to the interest of the former client, unless the former client gives informed consent, confirmed in writing.

o Substantially related has two prongs: can be the same transaction or legal dispute and material that is confidential would be divulged if you were to represent the new client and harms the former client

o There is a presumption in this rule to what substantially related means A former client is not require to reveal the confidential info learned by the lawyer in

order to establish a substantial risk that the lawyer has confidential info to use in the subsequent matter

(b) A lawyer shall not knowingly represent a person in the same or a substantially related matter in which a firm with which the lawyer formerly was associated had previously represented a client

o Conflicts can arise when a lawyer moves from one firm to another CHART ON PAGE 480!!!! Read chapter 8

THE NATURE OF CONFLICTS BETWEEN PRESENT AND FORMER CLIENTS• See chart on page 474-475• The main concept is page 480• Rule 1.9: a lawyer who has formerly represented a client in a matter shall not thereafter represent another

person in the same or a substantially related matter in which that person’s interests are marginally adverse to the interests of the former client unless the former client gives informed consent, confirmed in writing

• ***Comment 3: “substantially related” matters: if they involved the same transaction or legal dispute or if there otherwise is a substantial risk that confidential factual information as would normally have been obtained in the prior representation would materially advance the client’s interest in the subsequent matter.

• LOOK at substantially related analysis on page 483• Keep 1.7 in mind....• Try to make a factual determination of whether or not it would hurt the former client.• do not undo work you’ve done for a former client...• rules prevent you from switching sides (which is why we ask if it’s the same matter or substantially same)• So you have to look at two things: the past representation, and the current representation• Rules don’t really provide guidance about how long duty to client lasts: See comment 7 to rule 1.9, then see

rule 1.6• Have you really terminated the relationship? Even if you have had a long-term client yet have not done

work for them in the past year, are they still a client? Yes if you have not formally concluded representation

Westinghouse v. Gulf page 490 Price-fixing case in which Gulf We previously represented Gulf and he says that we should be disqualified because it is a substantially

related mattero Patents, leases, title disputes, counseling for issues, lobbying for Gulf

Gulf argues that it is substantially related to previous matter Gulf said it share confidences in the property Court held that it is reasonable that Bigbee Firm would have received confidential info but the court

declined disqualification because it was irrelevant to the current litigationo Seventh Circuit (Ct of Appeals) reversed saying Bigbee firm should be disqualified

What is materially adverse? Page 492 Not the same as direct adversity in Rule 1.7b

Page 35: Professional Responsibility Outline

Less restrictive and looser

• Problem 8-1: Keeping in touch:• You performed work for Almond 5 years ago and now a computer service company wants to hire firm to sue

Almond• Ask if this is a continuing relationship? Is the newsletter seen as giving advice?? Probably not a present

client!• Firm has tried to contact Almond over the years to see if he needed more services, etc., but he has no contact

with firm• Almond is definitely a former client (It was Brief, Discreet, and even when the former client was solicited to

renew the relationship, the former client always declined)• We must apply Rule 1.9• This was a tax opinion issue for Almond before, BUT the factual information you might discover would be

relevant to the bill collection by the new client in suing Almond (see Comment 3 to Rule 1.9). 1.9 is PREDICTIVE because you want to protect the client from a conflict...

• Under the Restatement, the client’s reasonable understanding of the scope of representation controls. So you could look to this...

• Factors:• Brief, One issue, Long time ago, The newsletter wasn’t advise

• In trying to determine if you have a former client, Look at:• Scope of rep, Length of years, What the client expected

• Fact specific determinations of the lawyer’s representation: IS the client serious enough that the lawyer cannot go on representing without obtaining consent?

• Hot-Potato • Lawyer cannot fire a client to get rid of a client and retain the more lucrative client. But there are exceptions:

• Material adverse is more narrow: you look at the likelihood of risk and the seriousness of its consequences.

EVALUATING SUCCESSIVE CONFLICTS:• Are the conflicts serious enough that the lawyer may not go forward w/o client’s consent?• SAME MATTER: anything that is the subject of representation: litigation, a transaction, a subject on which

a client requests advice (or a dispute over document that lawyer drafted)• SUBSTANTIAL RELATIONSHIP: Focus is on exploring the facts of a problem and on what a lawyer

might have learned during the first matter that could be used adversely to the former client in the second.

• Problem 8-2: The District Attorney• You are D.A. and represented a child a long time ago in an auto accident. Same boy, now 17 yrs old is

charged with murder. the defense wants you to withdraw b/c you previously represented him when he was 5 yrs old in the auto accident.

• Should you withdraw? YES, in real case, the attorney had to withdraw. He had information from his prior representation of the kid that could have included relevant medical information. You could have used med info to discredit his case.

• Is this the same/substantially related? Material adverse? if yes, only way to proceed is with consent of the client.

• Real case: prosecutor was disqualified• Rule 1.9!

• Maritrans v. Pepper, Hamilton & Scheetz• Cmt 6 to 1.7*** • Attorney Messina, a partner at Pepper firm, represented Maritrans, a business, for a long time.• During their relationship of over ten years, Maritrans shared a lot of confidential information about their

business practices, as well as info about their competitors• Pepper then began representing several of Maritrans’ competitors• Maritrans found out and objected, but Pepper said this was a “business conflict” not a “legal conflict.”

Page 36: Professional Responsibility Outline

• Pepper and Maritrans negotiated a deal that Pepper would not take on representation of any other competitors, especially Maritrans’ largest competitor, of Maritrans agreed to let Pepper continue being their attorney and would not represent any more than the four companies Pepper was already representing. In addition, the Pepper attorney with the most knowledge about Maritrans would not counsel Maritrans, but instead would represent the other four companies, and then 2 different attorneys would represent Maritrans.

• Chinese wall: attorneys on one side of the wall do not discuss their respective representations with the attorneys on the other side.

• The firm thought perhaps this was just an economic conflict, not a materially adverse conflict. • Then Messina parked the largest competitor with another attorney at another firm, and was working on

bringing that attorney over for partnership to Pepper firm.• Comment 6 to 1.7• Three main points:

• Injunctive relief: stopping something by preserving the status quo. • Fiduciary duty:• Conflict: can you really consent to these situations?• What do “substantially related” and “material adverse” mean?

• Rule 1.9 doesn’t give us specifics but if you look at the comments, 1, 2, 3, there’s guidance on how to look at these terms.

• Here, there was more than a mere economic conflict.• The trial court granted a preliminary injunction. The superior court reversed. This court reverses the superior

court.• Court says there are times when the danger of revelation of the confidences of a former client is so great that

injunctive relief is warranted. This is the case here. • There is a civil liability to support an injunction here. • If “fiduciary duty” has any meaning, we can’t allow these types of situations. This is conflict that should have

been enjoined. • DISSENT: consent was given here, and there was a contract under agreed terms of representation, which

were not breached!• First ask if it’s substantially related, then ask how it affects the client, and if it’s material adverse

• DIFFERENCE BETWEEN 1.9A AND 1.9B:• 1.9A: ASK WHETHER THE ATTORNEY COULD HAVE ACQUIRED CONFIDENTIAL INFO IN THE

FIRST REPRESENTATION THAT MIGHT BE USED ADVERSELY TOT HE FORMER CLIENT’S INTERESTS

• 1.9B: ASK WHETHER THE LAWYER ACTUALLY ACQUIRED MATERIAL CONFIDENTIAL INFORMATION. (WHERE THE PERSON OR SUBSTANTIALLY RELATED MATTER WAS REPRESENTED BY THEIR FORMER FIRM)

• To know if a lawyer has acquired material confidences, one must analyze the specific facts relating to the lawyer’s access to or information about the relevant matter. Comment 6 after Rule 1.9 directs analysts to use certain assumptions

• Comment 8:• If you have not acquired material confidences, you don’t need consent.• In any situation in which there is a substantial risk that the lawyer would normally have obtained confidences

in a former matter that could be used adversely to the former client in a subsequent matter, the lawyer may not be able to handle the second matter unless the former client consents.

• 1.9C: bars revelation of confidences received from former clients and prohibits the adverse use of such confidences.

• Problem 8-3: Dysfunctional family:• Henry and Joseph co-own HER, Inc. and HER Fashion, Inc.• There is a dispute and the ownership is divided.

• Henry own HER Inc.- Joseph owns HER fashions• Attorney resigns, continues to represent HER fashions

Page 37: Professional Responsibility Outline

• HER Inc. is still using the logo. Henry’s new attorney wants you to withdraw.• Joseph wants you to continue representing him for HER fashions.• Do you have to withdraw?• ANALYSIS of Rule 1.9a: (prohibits using information of one client against the other)

• Same? Substantially related?• “Materially adverse”?• Consent (Which in this case is not an option)

• The general counsel would have substantial involvement with the client• Real case: attorney was disqualified:

• “The obligation of attorney to not misuse info obtained in representation is to protect the client.”• Substantive overlap in work performed for these clients

• You have too much info, so you must withdraw. He can still rep the corp. generally, just not in this action.

Imputation of Former Client Conflicts to Affiliated Lawyers Allows for screening clients Rule 1.10 Imputation of Conflicts of Interest: General Rule

o 1.10(a)- Lawyers in a firm (considered one lawyer) shall not knowingly represent a client that any one lawyer in the firm would be prohibited from doing so under Rule 1.7 (Current client) or Rule 1.9 (Former client); unless…

(1) The prohibition is based on “Personal Interest” and does not present a significant risk of materially limiting the representation of the client by the remaining lawyers in the firm (personal interest is no conflict); OR

(2) The prohibition is based upon Rule 1.9a and b, and arises out of the disqualified lawyer’s association with a prior firm, and

(i) The disqualified lawyer is timely screened from any participation in the matter and is apportioned no part of the fee therefrom

o (This says that even if there is a conflict, the client still can be represented by screening the conflict from the lawyer that has a conflict and that lawyer cannot participate or receive any funds)

o Screened means stop the spread of any conflicting information (Isolation) and have a timely procedure that is reasonably adequate under the circumstances

(ii) Written notice to former client, which shall include a description of the screening procedures employed; a statement of the firm’s and of the screened lawyer’s compliance with these Rules; a statement that review may be available before a tribunal; and an agreement by the firm to respond promptly to any written inquiries or objections by the former client about the screening procedures

(iii) Certifications of compliance (to show that law firm has complied) with these Rules and with screening procedures are provided to the former client by the screened lawyer and by a partner of the firm, at reasonable intervals upon the former client’s written request and upon termination of the screening procedures

o 1.10(b)- When a lawyer has terminated an association with a firm, the firm is not prohibited from thereafter representing a person with interests materially adverse to those of a client represented by the formerly associated lawyer and not currently represented by the firm (the conflict leaves with the former lawyer of a firm), unless

The matter is the same or substantially related to that in which the formerly associated lawyer represented the client; and

Any lawyer remaining in the firm has information protected by Rules 1.6 and 1.9© that is material to the matter

A disqualification prescribed by this rule may be waived by the affected client under the conditions stated in Rule 1.7

The disqualification of lawyers associated in a firm with former or current government lawyers is governed by Rule 1.11

Page 38: Professional Responsibility Outline

o Look at Comment 5

• Problem 8-4: The Firm’s New Partner• Blurry lines between actually being part of the same firm with attorney representing co-defendant.• This is not the same firm- shared information is a possibility though. There is no framework for preserving

confidentiality (the secretary is shared). • If the lawyers become partners, you can’t make an appearance • Collinwood would violate rule 1.9a• What if they became partners and wanted to rep clients in separate trials?

• NO- once they become partners, they have fiduciary duties to both clients and can’t attack each other.• 4: There would be material adversity there: each would be pointing fingers at each other, particularly in

appeal.• Representing current client after dropping one could lead to the use of confidential information to the

detriment of the former client.• What really happened in this case?

Problem 8-5 The Fatal Shot Chad C and Mojo K are two sole practitioners, both doing criminal defense work They share a suite of offices, but operate a separate business They share one secretary and talk about each others cases all the time Two defendants were charged of murder (Lubinski and Bellavia) One hired Chad C and the other was recommended to hire Mojo K Questions

o May Mojo K agree to represent defendant Bellavia? They hold themselves out as separate firms but there is no system to ensure

confidentiality Still possible but there are difficulties to whether Mojo K can represent other defendant

o Say that Mojo K can represent, what if Mojo cant make the arraignment date and asked Chad C to step in for her to plea not guilty (when both of their clients are pointing the finger at the other)

May Chad C ethically substitute for Mojo K at arraignment?o Question 4 on page 518

Walk through Rule 1.9 and Rule 1.10o Chart on page 519 is good

Lawyer Moves from Firm A to Firm B. The lawyer does not bring any clients with him.o To analyze conflicts the lawyer carries with him into Firm B based on confidences learned at Firm

A: look at Rule 1.9bo To analyze conflicts that other lawyers at Firm B may have because of the lawyer’s work at Firm

A: look at Rule 1.10(a)o To analyze conflicts remaining at Firm A as a result of work the lawyer did while there: Look at

Rule 1.10(b)

CHAPTER 9: CONFLICTS BETWEEN LAWYERS AND CLIENTS

LEGAL FEES:• Rule 1.5: FEES : is a soft rule: allows for much discretion (as long as the fees are reasonable)• No contingent fees in criminal cases• 1.5a: A lawyer shall not make an agreement for, charge, or collect an unreasonable fee or an unreasonable

amount for expenses. The factors to be considered in determining the reasonableness of a fee include the following:

• (1) the time and labor required, the novelty and difficulty of the questions involved, and the skill requisite to perform the legal service properly

• Lawyer shall not make an “unreasonable” fee. “reasonable” is determined by a number of factors: These factors are not exhaustive, but allow us to make a determination:

Page 39: Professional Responsibility Outline

• Time and labor required• likelihood that acceptance of the case will preclude other employment by attorney• fee customarily charged in locality for similar services• the amount involved and results obtained• time limitations imposed by client or the circumstances• nature and length of the professional relationship with the client• experience, reputation, and ability of lawyer/s performing the services• whether fee is fixed or contingent

• Cmt 1: each factor won’t be relevant in every case.• Contingent fees SHALL be in writing, signed by client• Cmt 7: division of fees: client must agree to the division

• Brobeck, Phleger & Harrison v. Telex: • attorney filed petition for cert, other party withdrew, and the atty charged 1 million dollars• the written memo specified that no fee would be paid if petition was NOT filed and there was no settlement....• Court says it’s not unconscionable:

• the client was a multimillion dollar company and they wanted to hire a highly reputable firm.• Fee was not unconscionable, and fee was not unethically high.

• The memorandum was very ambiguous:• didn’t disclose scope of representation• didn’t break down fees

• In the Matter of Fordham: • Public censure, excessive fees• Timothy hired Fordham to represent him for OUI, driving w/ suspended license, speeding, etc.• clients should not have to pay for the education of their attorneys• Why is this excessive and how do we compare it with Brobeck?

• Brobeck:• sophisticated client

• Fordham:• young client

• Should the sophistication of the client be a factor in determining a reasonable rate?• The results are related to what type of case it is• We dont want attorneys learning on the job and then billing their clients for it.

• Brobeck and Fordham came out completely different!• Brobeck was a highly experienced and reputable attorney and the result was in favor of that side so the

court held that the fees of 1 million dollars was not unreasonable• Fordham court held that the 50,000 dollars in fees was unreasonable. Looking at the bills, it was not

expressed exactly what the client was paying for. It took the attorney far too much time (over 200 hours) to prepare this case when this case was not that difficult (client should not have to pay attorney’s education on the issue)

• Problem 9-1: An Unreasonable Fee • Apartment suffered 70,000 dollars damage from flood. Tenant sues LL to get payment. • Tenant hires attorney at 250/hour, he ends up working so much that the bill is 60,000• They never really discussed how much time should be spend on case (1.2: scope of representation). there is

ambiguity- the client says go and get the result, the atty goes and does the work and spends an inordinate amount of time on it. He worked really hard and did tons of work!

• Can you justify the 60,000 bill? this is a close case.• Maybe this could have been alleviated with communication: the attorney needed to talk about scope of

representation 1.2, communication, 1.4

Page 40: Professional Responsibility Outline

• cmt 5 to 1.5: its proper to define the extent of services in light of client’s ability to pay. should not exploit a fee arrangement based primarily on hourly charges by using wasteful procedures

• Rising Prices: Problem 9-2: Modification of fees • Can’t just raise fees• See 1.5d: any change in basis or rate of fee must be communicated to client. The fee must still meet

“reasonableness” requirement• Problem gives 4 options for actions to pursue in a rate increase. they would all pass under 1.5d

• Include notice in your next month’s bills• Send letter to all clients notifying them of the reasons and when the prospective increase will take effect• Send letter but also ask clients to assent by returning and signing letter• Maintain present rate for current client and charge new rate for new clients (most conservative, most

advantageous to clients, but not for you. you may not be able to pay for your bills)• Maybe you could take a mix of all three, and keep the current rate for clients that object, But this raises

fairness issues! so it could be problematic.• the first two seem like the best solutions• you should look at your contracts

NCBEX.ORG: Online Practice Exam

1.5: Division of fees between lawyers not in the same firm:• You can divide fees if:

• It’s proportional (to the work done)• Client consents in writing• And the fee must be reasonable.

• Comment 7: referral fees are allowed if you take ethical/financial responsibility for the action • A referring lawyer can be disciplined or sued if they don’t keep up with itRegulation of hourly billing and billing for expenses:• A lawyer billing by the hour may not bill for more hours than she actually worked• Lawyers may round up to a MINIMUM billing increment• Don’t invent hours that were not really worked• No profit on costs: a lawyer may not bill for “overhead” or markup costs• No double-billing: a lawyer may not bill two clients for one period of time• No billing a second client for recycled work• No churning or running the meter: a lawyer may not do unnecessary extra work in order to justify billing

more hours• No billing clients or the firm for personal expenses or marking up expense receipts• Lisa G. Lerman, Scenes from a law firm:

• Article that anonymously describes billing practices at one law firm:• Churning: turning out hundreds of Fords instead of a couple Mercedes• Padding: Bill more- put more hours down to add meat to the billing rate• Invent work• “Magic number”: calculating how many hours a day you need to bill to reach the annual target to get

bonuses and vacations, etc.• Cranking out forms: • Easy billing: read what comes and bill for it• Triple billing: if you can spend one hour doing three things, charge 3 clients for the hour• Billing off secretaries:• “Fifteen hours of paralegal time”: if the paralegal did more than the 15 hour max allowed for paralegals,

the remaining hours would be turned into attorney hours.• “Hit for .5”: if you wake up in your sleep and think of a case, hit .5 hours.• Thwarting the auditors: transferring hours on heavy days to hours for lighter days

Page 41: Professional Responsibility Outline

• Billing envy:

1.5 (c): Contingent Fees:• Are the lawyer and client’s interests truly aligned with a continent fee?• There is no maximum percentage for contingency fees: they just have to be “reasonable”• Contingent fees must be in writing (to explain details to the client, so they understand how it will operate-

will the fee come before/after expenses are subtracted, etc.)• In writing• Signed by client• State the method by which the fee is determined• Including the percentage/s that shall accrue to the lawyer in event of trial, appeal, etc.• Clearly notify client of any expenses for which the client will be liable whether or not the client is the

prevailing party• After the matter, attorney should provide written statement with the outcome of the matter and showing how

the fees were determined

Financial assistance to client:• RULE 1.8(e): you can’t give financial assistance to client in connection with pending or contemplated

litigation except:• You can advance court costs and costs of litigation, reimbursement will be given contingent on the outcome

of the matter• For an indigent client, you can pay their court costs and litigation costs

• RULE 1.8(i): a lawyer may not acquire a proprietary interest in the subject matter of litigation the lawyer is conducting for a client except permitted liens and contingent fees.

• if the lawyer purchases a claim from a client and ceases to represent them in collection of that claim, the transaction no longer violates Rule 1.8(i)

• Problem 9-3: Impoverished Client: • Your client is suing employer for firing him-age based discrimination• Employer says he was just unable to perform duties• You think he could win and might recover 50,000 in damages• He has financial problems: pending eviction, no savings, • You can pay litigation costs if they’re to be paid back by settlement• 1.8e

• Comment 10 • You can’t do this:

• We don’t want clients living off litigation• We don’t want the attorney to have a proprietary interest

• Real case:• A lawyer may pay client’s living expenses to avoid the client having to settle unfair claims (only in DC)

• You can help in other ways: help client settle with landlord, set him up in a shelter home, secure a loan against settlement for the claim....

1.8(d): Publication rights:• After the case is over, you can have publishing interests, but not while the case is ongoing.

Advance payment of fees and nonrefundble retainer fees:• 1.16: you must return any unnused fees• Page 575: • It’s controversial to require a nonrefundable advance payment from an individual client with whom the

lawyer has no prior relationship• “Classic retainer” or “general retainer”

Page 42: Professional Responsibility Outline

Fee Disputes:• Rule 1.8(h)(1): You can ask each client in advance not to sue the lawyer for malpractice, no matter what!

The client MUST be INDEPENDENTLY represented about this type of agreement.• 1.8(h)(2): Cannot settle claim or potential claim for such liability with an unrepresented client or former

client unless they are advised in writing that they should seek independent legal counsel, and give them time to do so.

LLP:General form of firms now: Limited Liability Partnershiptakes away general vicarious liability of partners: you’re still liable for your actions, and people you oversee, but not of your partners.

Fee arbitration:• when you continue to bill but your client is not paying, you have to remember client confidences• Lawyer CANNOT obtain liens on client property. back to 1.8(i): you don’t have lawyers to have

proprietary interests in client. HOWEVER, you CAN obtain lien if there is also a conract for it, b/c it then constitutes a business transaction.

Division of FeesRule 1.5(e)

A division of a fee between lawyers who are not in the same firm may be made only if:o The division is in proportion to the services performed by each lawyer or each lawyer assumes

joint responsibility for the representationo The client agrees to the arrangement, including the share each lawyer will receive, and the

agreement is confirmed in writing; and o The total fee has to be reasonable

Entering into deal with a client; ask these questions Fairness: Terms with client have to be fear Have to be explained to the client clearly and in writing Has to be able to seek independent counsel on the matter and have reasonable time Have to give informed consent in writing

Lawyer as custodian of client property and documents• Client trust account s

• NO CO-MINGLING!!!!• You have to keep the client’s property separate.• Rule 1.15• You must keep complete records of the funds or other property for a period specified in state rules.

• Prompt delivery of funds or property: • When you receive a settlement check or other funds that should be paid to client, you have to notify the client

and make prompt payment of all funds due to client• What if there is a dispute about money or property in lawyer’s possession?

• 1.15(e) and (c): if there is a dispute about the amount of the fee, lawyer should distribute undisputed portions and keep the disputed part in the client trust account

• Creditors: page 549• Unless the creditor has a legitimate claim to the particular funds in the lawyer’s possession, the lawyer’s duty

is to the client, not the 3rd party• Business transactions between lawyer and client:

• RULE 1.8(a): three requirements:• See page 553.• TEST: questions to ask on page 554:

• Problem 9-4: Starting a Business

Page 43: Professional Responsibility Outline

• If you go overboard with disclosure, he might back out of the deal• Are there any special disclosures that are needed?• Who represents whom? Through the transaction, he represents himself, after the transaction, he represents the

corporation. You need to disclose that you represent your own interests for the purpose of THIS transaction. If any conflict should ultimately arise, I represent the corporation. In order for Lopez to give consent, you have to disclose your interest, and your status as representative of the corporation

• 1.8a• What to do:

• Tell him he should probably get other representation in writing and give him opportunity to do so• Explain that you will represent the company not him• Fair and reasonable terms, client has reasonable opportunity to go over writing with another attorney,

have all terms in a written agreement, counsel only represents the corporation and not the other partner• Real case: Rhode Island (attorney was former Lt. Governor) Attorney did not make disclosure so the friend

just thought the attorney was a lender, not really a shareholder. Ct held attorney breached his fiduciary duty to corporation- and the President of Corporation could have sued (which he did not do). Ct concluded it was too late for President of Corporation to undo transaction but he allowed President to buy out the attorney for his 20% at the date action was filed. No disciplinary action was taken.

• In disclosure, there HAS to be notion that attorney represents the corporation, not the individual.• GIFTS FROM CLIENTS:

• No substantial gifts allowed unless person is related to the attorney (family members, etc.)• Because fiduciary relationship and we don’t want there to be any pressure client to do

anything and vice versa• Can’t prepare instruments for clients giving gifts to the lawyer of his relatives (but you can prepare one for

your close relative)• Rule 1.8(c)

• SEXUAL RELATIONSHIPS WITH CLIENT: (Rule 1.8(j)) • Not allowed unless the sexual relationship exited before the lawyer-client relationship began.• Confuses attorney-client privilege• Judgment would be overcome• If relationship starts, must get independent counsel (COMMENT 17)

• DIFFERENCE b/w PERSONAL CONFLICT AND IMPUTED CONFLICT (1.10)*** • personal conflicts cannot be imputed to members of the firm. they do not present significant risks of

materially limiting the representation of the client by the remaining lawyers in the firm.

Chapter 10: Conflicts Issues for Government Lawyers and Judges: RULE 1.11• Rule 1.11 is the only rule that allows screening• Talks about ethical codes of judges (governed by rules of professional responsibility of course)• Federal Judges are governed by code of judicial ethics, but NOT the Supreme Court judges

• Now a movement to bring the SCt under more scrutiny• Canon of Judicial Conduct:

• Comment 4 highlights the goal of the rule (balance interests) We want people to move back and forth, but we don’t want attorneys to use info to help or hurt clients.

Government Lawyers Rules under 1.11 is less stringent than rule 1.9 Rule 1.9

o Successive Conflict Rule 1.11

o Government Lawyerso 1.11(a) Former Government (Public) Employee or Officer entering into private practice

Is subject to 1.9(c) which is the protection of former client confidential information and cannot represent a potential client that may be adverse to a former client)

Page 44: Professional Responsibility Outline

Shall not otherwise represent a client in connection with a matter in which the lawyer participated personally and substantially (direct and significant involvement) as a public officer or employee, unless the appropriate government agency gives its informed consent (confirmed in writing)

We don’t want blanket exclusion just because you worked at a former govt agency

o 1.11(b) Screening a lawyer (don’t need consent) When a lawyer is disqualified from representation under paragraph (a), no lawyer in a

firm with which that lawyer is associated may knowingly undertake or continue representation in such a matter unless

The disqualified lawyer is timely screened from any participation in the matter and is apportioned no part of the fee therefrom, and

Written notice is promptly given to the appropriate government agency to enable it to ascertain compliance with the provisions of this rule

o 1.11(c) Confidential Government Information A lawyer having information that the lawyer knows (actual knowledge) is confidential

government information about a person acquired when the lawyer was a public officer or employee, may not represent a private client whose interests are adverse to that person in a matter in which the information could be used to the material disadvantage of that person

Confidential govt information means information that has been obtained under governmental authority and the govt is prohibited by law from disclosing to the public or has a legal privilege not to disclose and which is not otherwise available to the public

A firm may undertake or continue representation only if the disqualified lawyer is timely screened from any participation and is apportioned no part of the fee

o 1.11(d) Former Private Lawyer who is now a Government (Public) Lawyer Except as law may otherwise expressly allow, a lawyer currently serving as a public

(govt) officer or employee: Is subject to Rules 1.7 and 1.9; and Shall not:

o Participate in a matter in which the lawyer participated personally and substantially while in private practice or nongovernmental employment, unless the appropriate government agency gives its informed consent, confirmed in writing; or

o Negotiate for private employment with any person who is involved as a party or as lawyer for a party in a matter in which the lawyer is participating personally and substantially, except that a lawyer serving as a law clerk to a judge, other adjudicative officer or arbitrator may negotiate for private employment as permitted by Rule 1.12b and subject to the conditions stated in Rule 1.12b

o 1.11(e) as used in this rule, the term “matter” includes: Any judicial or other proceeding, application, request for a ruling or other determination,

contract, claim, controversy, investigation, charge, accusation, arrest or other particular matter involving a specific party or parties, and

Any other matter covered by the conflict of interest rules of the appropriate government agency

• Relate to 1.9(c)• Attorney is disqualified if they were involved personally and substantially, unless they have consent. If they

don’t have consent, the firm can still proceed with representation if the conflicted lawyer is screened and does not earn specific fees from the work.

• 1.11 doesn’t really define “personally and substantially” so the rules look to federal regulations for guidance

Page 45: Professional Responsibility Outline

• Personally: directly (includes the participation of a subordinate when actually directed by former Government employee in the matter

• Substantially: the employee’s involvement must be of significance to the matter, or form a basis for a reasonable appearance of such significance to the matter, or form a basis for a reasonable appearance of such significance. It requires more than official responsibility, knowledge, perfunctory involvement, or involvement on an administrative or peripheral issue

• “Confidential government information”: Comment 8• “Confidential government information”:

• The information that triggers the rule must have been obtained through governmental power (such as threat of subpoena)

• The govt must be barred from disclosing it or entitled to assert a privilege against disclosure, AND• The info must not be in the public domain

• 1.11(a) is the govt lawyer who was personally and substantially involved. We discussed consent• Basis for disqualification of former government lawyers:***bottom of page 570

• Subsequent work could involve use of “confidential governmental info” about a person known to the lawyer in a way that would materially disadvantage that person (This CANNOT be cured by consent) (rule 1.11(c)-type 2) **Bottom of page 570

• PROBLEM 10-1: Abraham Sofaer was a professor at Columbia Law, a fed dist. judge, and then the general counsel to the US Dept. of State (he developed the legal justification for economic sanctions against Libya) and for the US air raid on Tripoli that experts believe the Libyans sought to avenge with the Pan Am massacre. After he left govt, he went to private practice and agreed to represent an attorney from Libya about bombing of Pan Am 103.

• this attorney should have conferred with the state dept.• Look at comment 10 to 1.11• real case: he got semi-public reprimand. he appealed but it was affirmed.

• What’s the difference between 1.11a (screening, confidential info, consent) and 11.d???• 1.11d: How government lawyers should evaluate conflicts that arise because of work that the lawyer did in

a previous private law job (former clients).• The lawyer had to get a special license to rep Libya (because of an embargo he put in place). if he never

served in gvt there would probably not have been a problem. possible problem: he definitely participated personally and substantially in the matter of bombing. in the end he received semi public admonishment which he appealed. it was affirmed that he should be censured. he was not allowed to represent even though he had previously voluntarily withdrawn in response to public response. his involvement was a matter under 1.11.

• 1.11 comment 10: for purposes of paragraph e of rule, a matter may continue in another form in determining whether 2 particular matters are the same, consider:

• extent to which the matters involve the same basic facts• the same or related parties• the time elapsed.

• the atty failed to ask: the larger question:• how will this be perceived?• how will the perception affect the client?

CODE OF JUDICIAL CONDUCT:Four Canons:• 1. Impartiality: A judge shall uphold and promote the independence, integrity, and impartiality of the

judiciary, and shall avoid impropriety and the appearance of impropriety

• 2. Competence and diligence as a judge: A judge shall perform the duties of judicial office impartially, competently, and diligently

• 3. Undivided loyalty: A judge shall conduct the judge’s personal and extrajudicial activities to minimize the risk of conflict with the obligations of judicial office (including full disclosure)

Page 46: Professional Responsibility Outline

• 4. A judge or candidate for judicial office shall not engage in political or campaign activity that is inconsistent with the independence, integrity or impartiality of the judiciary

• Judges cannot be in discriminative organizations. This is a violation of the third canon. (sexual orientation has just recently been added as a type of prohibited organizational affiliation)

• Page 584: Scalia scenario: He argued that USSC judges are different- if a judge recuses himself, there’s an even number (4 to 4) and issues must be decided. Ultimately the issue was decided 7-2, in favor of cheney.

• was there a conflict?

• 10-2: this was an actual case• underlying allegation that judge is in the pocket of one of the parties in the suit.

Disqualification of Judges• Problem 10-2: A trip to Monte Carlo:

• You represent Apex, company that lost in lawsuit against another company.• You appealed, and it was overturned- one of the judges had taken a vacation with CEO of Apex during

pending of the appeal. • Now the other company’s lawyer files motion asking the judge to withdraw his vote and recuse himself from

participating in any further proceedings. If he does withdraw and recuse, the vote might be tied. • Rule 2.11 in the Model Code of Judicial Conduct states that a judge shall disqualify himself or herself in any

proceeding in which the judge’s impartiality might be reasonably questioned. He should disqualify himself even if no party files a recusal motion

• REAL CASE: two judges recused themselves- even after the re-vote, Apex won. The recusal didn’t really help anything.

• As for the lawyer, you should probably oppose the motion.

• Problem 10-3: Judge’s Former Professor: • You are an attorney for Ahr, a govt official who is a defendant for sexual harassment. You client used to be a

law professor. the judge in the case used to be client’s student.• Should a motion to recuse be filed? You don’t want to do that unless you really have to. You have to really

know what the relationship b/w judge and client is.• If the motion requesting recusal is filed, should the judge disqualify herself? It’s a maybe- See Canon 3• See Canon 2B: prohibits judge from allowing “family, social, political, or other relationships to influence the

judge’s conduct or judgment”• Canon 3B(7)(b) allows judge to obtain the advice of a disinterested expert on the law that applies to a

particular matter if the judge notifies the parties as to who is consulted and of the substance of the communication and allows the parties to respond.

• This is the real case of bill Clinton.

RULE 1.12• Rule 1.12(a)- judges who have been personally and substantially involved as a judge

• A lawyer shall not represent anyone in connection with a matter in which the lawyer participated personally and substantially as a judge or other adjudicative officer or law clerk to such a person or as an arbitrator, mediator or other third-party neutral, unless all parties to the proceeding give informed consent, confirmed in writing (judge will be screened)

• Rule 1.12(b): Page 641: You can’t negotiate for employment with any person who is involved as a party or as lawyer for a party in a matter in which the lawyer is participating personally and substantially as a judge or other adjudicative officer. A law clerk to a judge or other officer may negotiate for employment with a party or lawyer involved in a matter in which the clerk is participating personally and substantially, but only after the lawyer has notified the judge/officer.

• Law clerks are permitted to apply for jobs, even with parties/lawyers involved in matters that are pending before their judges, but only after they notify the judge/officer.

• Rule 1.12(a)

Page 47: Professional Responsibility Outline

• “personal and substantial participation” standard is the same as that used for conflicts of gvt lawyers in rule 1.11

Chapter 11: Lawyers’ Duties to Courts:• There are boundaries to loyalty to the client. • On one end, you have “hired guns”, on the other, you have “officers of the court” mentality.• Rule 3.1: A lawyer may not file frivolous lawsuits- but we don’t know what frivolous is

• Comment 2: says facts need not be fully substantiated (this is not a clear black letter rule- it’s factually determinative). the lawyer may use discovery to develop vital evidence

• On the other hand, the lawyer must do basic investigation• FRC 11(b): similar to 3.1 but more detailed: A party’s legal theory must be “warranted by existing law or

by a nonfrivolous argument for the extension, modification, or reversal or existing law or the establishment of new law.” Factual assertions must have “evidentiary support or, if specifically so identified, by likely to have evidentiary support after a reasonable opportunity for further investigation or discovery.” This standard is subject to widely ranging interpretation

• Footman v. Cheung (type notes for pages 600-• Parker v. Vigo County School Corp• Jimenez v. Madison Area Technical College• Differences b/w Rule 3.1 and FRCP 11: see page 602• Penalties applied to lawyers who bring unsubstantiated suits:

• Rule 11 penalties• Attorney’s fees• Liability for malicious prosecution

• Problem 11-1: Visit from Paula Jones:• You should investigate and question people at hotel who might have witnessed things• track down bodyguard• maybe reach out the president- just enough to corroborate what your client is telling you• can you file a claim now? Once you can find some corroboration, you can file the claim (b/c discovery will

allow you to find more later)

Rule 3.1 Meritorious Claims and Contentions A lawyer shall not bring or defend a proceeding unless there is a basis in law and fact for doing so that is

not frivolous, which includes a good faith argument A lawyer for the defendant in a criminal proceeding, or the respondent in a proceeding that could result in

incarceration, may nevertheless so defend the proceeding as to require that every element of the case be established

You can work and develop a claim; discovery will help when you are trying to figure out the full details about your client’s case

Sanction here would be bar disciplinary action, where as Federal Rule of Civil Procedure 11 is a punishment from the judge

Lawyer’s Duty to Courts/Obligation to Non-Clients• Candor towards tribunal- RULE 3.3:

• (A) A lawyer shall not knowingly:• Make a false statement of law or fact or fail to correct a false statement of material fact or law previously

made to the tribunal by the lawyer• Fail to disclose to the tribunal legal authority in the controlling jurisdiction known to the lawyer to be

directly adverse to client’s position• Offer evidence that the lawyer knows to be false. if lawyer, his client, or witness called by lawyer has

offered material evidence and attorney comes to know of its falsity, attorney has to take reasonable remedial measures, including, if necessary, disclosure to tribunal

• (B): A lawyer who represents a client in an adjudicative proceeding and who knows that a person intends to engage, is engaging or has engaged in criminal or fraudulent conduct related tot he proceeding you shall take reasonable remedial measures (including if necessary, disclosure to the tribunal)

Page 48: Professional Responsibility Outline

←• (C): The duties stated in (a) and (b) continue to the conclusion of the proceeding, and apply even if

compliance requires disclosure of information otherwise protected by 1.6

• (D): In an ex parte proceeding, a lawyer shall inform the tribunal of all material facts known to the lawyer that will enable the tribunal to make an informed decision, whether or not the facts are adverse

• Rule 8.4 (c): Misconduct• This is what really got President Clinton• Misconduct is Broad- It builds on candor toward tribunal, but it applies to ALL conduct.• when we talk about lawyer’s interaction with other people, we’re pointing to

• 3.3 candor toward tribunal• 4.1 truthfulness in statements to others• 8.4: misconduct.

Rule 1.11 Special Conflicts of Interest for Former and Current Government Officers and Employeesa. “Matter” “personally” and “substantially”—consent of governmentb. Screening, give notice to appropriate governmental entityc. Confidential government info—have to have actual knowledge—cannot use this infod. Former private lawyer who is now a government lawyer is impacted by 1.7 and 1.9

• Nix v. Whiteside: (“There was something metallic) • Does revelation of client perjury undermine effective assistance of counsel?• NO. There is no 6th amendment violation• It is elementary that a right does not extend to testifying falsely.

• How can we ever know whether a client plans to give false testimony?

CANDORRule 3.3, comment 5, 10, 13Rule 8.4 (Misconduct) dishonesty, fraud, deceit, prejudicial to administration of justiceRule 4.1 Truthfulness in statements to others (comment 3)

• Problem 11-2: Flight from Sudan: • You represent man seeking asylum from Sudan. He says he worked for an underground newspaper,

Democracy. You are doing work to corroborate his info, and you actually end up getting in touch with the former editor in chief of the Democracy. he has never heard of your client.

• Should you tell your client what you discovered? Yes: Communication 1.4a3: keep client informed about important info. You should tell your client about the conversation

• Can you file the affidavit? Yes, it’s reasonable to still believe your client• Do you ask to withdraw? Not yet. you really want to talk to the client first and figure out what’s going on.• Did you violate rule 1.6 by contacting Al-Parah? No, there’s an implicit authority to investigate for your

client. Your client permitted you to investigate and reveal info, so you’re not revealing client confidences.• You haven’t made a mistake by doing your job too well. Maybe you’re doing your job too well- if you find

out all this info, you may have to act on it. but probably not.• You don’t have enough to say this is a lie. • REAL CASE: This was a law clinic case. The students were on the verge of filing the affidavit when they

found out about the information. They confronted him and he insisted the editor was mistaken, and the students were about to withdraw. The client fires the students and gets new attorney- the students didn’t know what to do - Luckily, the new attorney requested only the documents that had been filed in court at that time. Client got political asylum.

• See COMMENT 8: Lawyer’s knowledge that something is false can be inferred from the facts.

• Problem 11-3: Flight from Sudan, Scene 2:

Page 49: Professional Responsibility Outline

• This is NOT material under 3.3(a)(3)—not going to affect this case • Real case: Government attorney found out after end of case and was livid: judge rejected to rehear the case.

Judge says it was NOT material and declined to reopen the case.• Court refused to re-open case because the witness’s testimony was not relied upon. • There may be some collateral damage to the legal clinic’s reputation• What about the reputation of the law clinic and the attorney?

• OJ CASE: • Cochran, OJ’s attorney redecorated OJ’s house before the jury came over to his house.• Was this despicable?• I don’t think so: it’s not like you’re concealing or replacing material evidence. You’re not changing the facts

of the alleged crime. Judge’s disclaimer:?

• Problem 11-4: Drug Test • Your client had to be tested for Meth, and he tested negative, but the test showed traces of marijuana. You

only want the Meth results for the judge, and the lab tech says they can resend you the results with only the meth results- when you get it, you realize she just cut off the bottom portion of the paper.

• What should you have done when you got the results? Could you just have told your client to go somewhere else and ONLY get tested for meth?

• RULE 3.4: can’t alter or destroy document that has evidentiary value• when evidence is submitted, it holds a stamp of authenticity on behalf of the lawyer• Most attorneys would say it’s okay to get a new test. • It’s ethically okay too

• It’s competitive in nature• If they request a full test however, that’s different and you must comply• What if you submit the partial report that the tech sent you. If you move to submit the report for court, you’re

implying it’s complete. Making it look good would probably require some careful doctoring too.• What if the court asks you if this is the entire report? how do you respond?

• If you just say “yes your honor” - that’s close to perjury• “That’s what I have, that’s what I asked them to screen for”:

• REAL CASE: Attorney presented the partial documents. He gave the second response (that’s what I have, that’s what I asked them to screen for). The judge was suspicious of the document. Judge ordered new tests- tests came back negative for everything. The attorney was so nervous however, that he revealed everything- judge reported attorney to the state that brought criminal contempt charges against the attorney. He negotiated it down to a civil charge and paid a $100 fine.

• Attorney violated 3.3: failed to disclose a material fact to the court.• Answer: You should get a new report separately and turn that in• 8.4c and d: administration of justice• Go through the wrong answers on page 628

• Problem 11-5: The body double • REAL CASE: Illinois Sup Court: Atty didn’t give any notice, • Atty put a look-alike person on the stand, and got a Misidentification. then asked to clear the courtroom and

told the judge. • atty moved for directed virect- the defendant was acquitted• HOWEVER< the JUDGE was ticked off! and held atty in contempt: “This is a misrepresentation by the

inferences of the totality of conduct”. went up tto sup court and on 4-3 vote, the atty criminal contempt was upheld in state supreme court for deceiving the judge.

• “This type of behavior undermines the dignity of the court”• misrepresentation by inference• So it was a close cut question!

• Problem 11-6: Refreshing Recollection • How you’re supposed to interact with others

Page 50: Professional Responsibility Outline

• Other witnesses /plaintiffs tell you that the Superior told them their rates were lower than other companies• How far can you go with the witness that doesn’t tell you this?• Can you ask whether the rep made any statements to her to the effect that their rates were lower than those of

other companies? Yes! b/c you’re not supplying her with info that she doesn’t have• Can you tell her that the company settled other cases favorably b/c other borrowers remembered that the rep

said their rates are lower? This goes too far because you’re kind of telling her the client what to say. violation under 3.4b

• Can you remind her of what the rep looked like? No. You’re providing her with information she doesn’t have. you can show her photos like a lineup

• Anytime you tell someone/some entity what to say, there’s a problem• you can’t supply them with knowledge they don’t have. • Can you tell her what others have said? then when she says she remembers, can you tell her to be firm when

the testifies? Sure, this is all okay. YOu just can’t cross the lines of telling her what to say...• “refreshing recollection” vs. supplying information

• RULE 3.3• What does “counsel or assist” mean? Can you coach as long as it doesn’t induce false evidence

• RULE 3.4: Lawyer shall not unlawfully obstruct another party’s access to evidence or unlawfully alter, destroy or conceal a document or other material having potential evidentiary value. a lawyer shall not counsel or assist another person to do any such act.

• This rule does not prohibit all concealment or destruction of evidence but only “unlawful” concealment.• “Unlawful” means the lawyer already has some obligation to disclose it.

• add cases from page 635-40

• PROBLEM 11-7: CHILD PORNOGRAPHY: • you represented one of the most respected churches in town. the choir director worked for the church for 30

years. he loaned his computer to another church employee who saw child porn on the computer. he loanee informed the rector who notified you. The choir director admitted that he owned the stuff and then resigned. There has been no publicity and no charges filed. what do you do with the computer now that you have it?

• REAL CASE: Connecticut: He took the laptop and destroyed it. indicted under the obstruction of justice under the sarbanes-oxley statute. he eventually pled guilrty for not reporting a felony. this was really a stretch of sarbanes-oxley

• Unclear whether or not he really needed to report it but it may have been required.• Rule 3.4 Comment 2

• evidentiary discovery is very important procedural right- don’t destroy evidence• in some cases you can take temp possession but you might be required to turn over evidence to police,

etc.• the choir director got 5.5 years.

• Articles on page 645-50: • Austin Sarat: There is usually no consequence at all to an attorney for hiding documents. what matters is

winning the case and keeping the client.• PROBLEM 11-8: Damaging Documents:

• Should we turn over these smoking gun documents?• Pay attention to the definitional section • Can you send them 60,000 documents to overload them but not send them the other 2 bad documents? and

draft them a letter saying thatyou’ve complied?• legalistic rationales could cause problems. maybe you should produce everything-- and be overly thorough. • If you conceal them, you could say you’re not disclosing them and opposing counsel set the parameters of

disclosure• Real case: They concealed the documents and that was the risk they took. Ifyou conceal a document, it’s very

difficult for it to be uncovered. The opposing counsel caught wind of it and was going to re open. They settled. The firm was sanctioned. It was almost like it wasn’t a big deal though- the attorneys who were involved were not fired and at least one of them was promoted to partner.

• You have to have rational arguments to show why non-disclosure is “substantially justified”

Page 51: Professional Responsibility Outline

• so even though these atys were sanctioned, you almost have to ask yourself “where’s the harm?”• Look at the definition, respond, and then let them ask for more- afterall, that’s what requests for documents

are. We don’t really have a “dear doctor” letter, and the the other memo is not about Somophyllin• RULE 3.3-a-2:

• Prohibits lawyer from knowingly failing to disclose legal authority in the controlling jurisdiction that the lawyer knows is directly adverse to her client’s position, if an opponent has not already informed the judge of the adverse authority.

• You don’t have to disclose persuasive/or secondary authority. • Don’t have to disclose if it’s from different jurisdictions

• EX PARTE PROCEEDINGS:• lawyer shall inform tribunal of all material facts known to lawyer to enable the court to make an informed

decision, even if adverse to client. This duty overrides the obligation to protect confidences under Rule 1.6.• IMPROPER INFLUENCE ON JURY:

• Lawyer’s comments to the press:• Gentile case: 5-4 decision declaring that disciplinary rule was void b/c it was too vague: attys didn’t

know what they could or could not say. • RULE 3.6: trial publicity:

• b, 1-6: list of publicity that’s ok• b-7: specifically in criminal cases• c: allows lawyers make a statement that a “reasonable lawyer would believe is required to protect a client

from substantial undue prej effect of recent publicity.• d: if one lawyer in a firm can’t make a statement, neither can others in that firm• Comment 5: list of likely prejudicial topics• generally though safe to give press copy of docuemtns that have been filed in court.

• GENTILE CASE:• those clauses were too vague to determine what the attorney could say.

• Problem 11-9: LETTER TO EDITOR:• can you send this letter? Question is whether it would materially prejudice the proceeding.• start with 11-9: look at comment 5.• Rule 3.6: Comment 5: Factors 3 and 4 are really important: the evidence of lie detector test and saying your

client is innocent could be really prejudicial. In a courtroom these would be okay, but is different in public opinion.

• take out the :abominable” passage and the lie detecotr and innocence parts then submit it• Or you could have a citizen submit it- they have greater 1st amen rights• REAL CASE: lawyer submitted it as it was: publicly reprimaded (not high consequence b/c he had a great

reputation)• Subin Article:

• Professor Subin: on some level, lawyers are truth manipulators• In one case, he represented a defendant accused of rape. The def. admitted to Subin that he did rape the girl.

In trial, Subin tried to prove that the sex was consensual.• Can you cross-examine a witness in a harsh criticizing way, if you know the witness is being truthful?

• It would be misrepresentation to the court, but it would be a minor misrepresentation, not a major misrepresentation.

• If you were entirely making stuff up, it would be a different story, but if you’re simply putting the prosecution to the test, it’s proper. As long as youre not manufacturing informaiton, it’s okay.

• Subin’s recommendation: The defense lawyer is limited only to good faith challenges (so if a def. confessed, you would have to find a good-faith basis to pursue a defense).

←• Statements by Lawyers During Jury Trials:

• 3.7: Lawyer as witness: • A lawyer shall not act as advocate at a trial in which the lawyer is likely to be a necessary witness unless:

• testimony relates to an uncontested issue• relates to the nature and value of legal services rendered in the case, or

Page 52: Professional Responsibility Outline

• disqualification of the lawyer would work substantial hardship on the client• A lawyer may act as advocate in a trial in which another lawyer in the lawyer’s firm is likely to be called

as a witness unless precluded from doing so by Rule l.7 or Rule 1.9• Comments by lawyers appealing to racial or other prejudice of lawyers: Comment to rule 8.4 says that doing

so is a basis for discipline.• Rule 3.4: Fairness to Opposing Party and Counsel

• Page 672• Lawyers’ duties in nonadjudicative proceedings:

• Chart 676

Exam Review:Malpractice, what does that mean?Sort answers: based upon problems.Nothing about judicial conduct, but the rest of chapter 12 is free game• Judicial conduct: main concern of the 4 canons is impartiality• Chapter 12

• 8.4 general misconduct• 4.3• Emergency food stamps: can you have a paralegal deceive someone? it’s a lie, but on some level it’s a good

lie- it’s not material. the atty did identify himself after the supervisor got on the line• why should you misrepresent in this way?

• 8.4• 4.1

• the rules make a distinction between what is contextually true: whether or not something is statement of fact can be deducted from circumstancts

• 12-2:• atty should not misldea DeBello and should make clear who he represents• Fox may be asked to sign the written statement but could not say dibello needed him to

• 12-3:• 1.2d: can’t assist in activity you know is criminal• 4.3(a) shall not lawfully obstruct access to information• 4.4: did they receive it anadvertently?• 8.4 • So what is the right thing to do?

• destroy copies and never refer to them: if the client is willing to take the chance of being prosecuted, maybe the atty should too.

• Real case: atty showed the ex husband’s atty some of the docuemtns the client had obtained, explained that he knew the ex was probably a molester. On morning of trial to begin, father agreed to give sole custody to mother

• Had there not been a settlement, the documents may not have been admitted. But the little girls were probably old enough to testify at that point and maybe the documents wouldn’ have been needed

• 12-4: p 717: Is there an emergency exception to the rules?• can you pretend to be a public defender and negotiate ? this could save lives• maybe you can talk to him without obtaining any incriminating info (this is improbable)• 3.8: atty shouldnt seek from someone w/o representation that they waive their rights.• 4.1 violated• 8.4 Violated b/c he make false statements of fact and gave advice to someone whose interests were adverse• real case: atty suspended for 3 months for violating 4.1 and 8.4 Suspension was stayed for 12 months (court

felt that the atty took advantage of the defenant). the conduct dealt with dishonesty, fraud , deceit. He had to take CLE classes, pay costs of discipline hearing, and had to be supervised when meeting with unrepresented clients

• 12-5 page 727:• arguments jsutifying his statements:

Page 53: Professional Responsibility Outline

• maybe he’s just identifying a potential problem• there’ sno criminal law that says you can’t talk about unpopular clients• this notion of prejudice to the justice system is unconstitutionally vague (blatantly prejudical to

administartion of justice- what does thi smean?)• he should be able to express his own opinion afterall, he is a US citizen

• Arguments in favor of disciplining him:• This is a high ranking official with policy making power and he is articulating a view tha tmight have

national an dintl’ implications• he should be held to higher standard b/c status as high-rankingg vt official

• should he be reprimanded? disbarred?• Powell thinks public reprimand was appropriate• there was public uproar- ppl thought he was trying to intimidate firms• he was forced to resign: nothing reall yhappened and he became a fellow at the heritage foundation

• Third party interests that affect a lawyer’s behavior: see article on page 729:• “Other interests impacting how the law is practied”• there are community interests. • the rules say...you can accept unpopulat cases, doesnt mean you endorse it• but there are third party interests that affect these standards- this is the misconception of legal ethics.

CHAPTER 12LAWYERS’ DUTIES TO ADVERSARIES AND THIRD PERSONS

• COMMUNICATIONS WITH LAWYERS AND THIRD PERSONS • DECEPTION OF THIRD PERSON

• Rule 4.1(a): • In the course of representing a client a lawyer shall not knowingly.... make a false statement of

material fact or law to a third person....• False statements by clients:

• If the lawyer knows that her client is lying in her presence to someone other than a court, the attorney must withdraw from representing a client who is using the lawyer’s services to perpetuate fraud.

• even if no proceeding has commenced, and the client is not using the lawyer’s services to perpetrate fraud, the lawyer should interrupt the lying, take her client aside, and advise her to tell the truth. However, if the lawyer does not do so, or the client refuses to take such advice, the rules don’t require the lawyer to correct the record.

• Problem 12-1: Emergency Food Stamps: • Attorney had a paralegal call the welfare office pretending to be district manager so the welfare

office manager would answer the call. the atty then explained why his client had an emergency need to get food stamps immediately.

• Rules: 4.1 (is it material)? and Rule 8.4c• it’s certainly misrepresentation, but we think it’s okay.• comment 2 to rule 4.1

• Apple case, page 685:• the limited use of deception is not a violation of 8.4• However, in the Gatti case, the attorney was disciplined for violating a duty to maintain personal

integrity• is it okay for us to have this exception?

• RESTRICTIONS ON CONTACT WITH REPRESENTED PERSONS • Rule 4.2:

• See comments 1 and 7• In representing a client, a lawyer shall not communicate about the subject of the representation with

a person the lawyer knows to be represented by another lawyer in the matter, unless the lawyer has the consent of the other lawyer or is authorized to do so by law or a court order

• you do not want to undermine their atty-client privilege.

Page 54: Professional Responsibility Outline

• Harvard issue page 699: they were mere witnesses, and some of them had supervisory authority.• comparison of corporate atty-client privilege and rule 4.2: chart on 698

• focus on comment 7 to rule 4.7: interviewing employees of the corporation. There are people you cannot interview

• an employee who supervises, directs or reularly consults with the org’s lawyer concerning the matter

• has authority to obligate the organization with respect to the matter• is one “whose act or ommission in connection with the matter may be imputed to the

organization fo rpurposes of civil or criminal liability• Messing, Rudavsky, and Weliky v. President and Fellows of Harvard College:

• Rule 4.2• the rule used to have an admission rationale which was dropped when the rules were advised.

• why? it was overly broad- anyone would fit into the category (anything could be an admission if it falls within scope of your employment)

• court rejects admission standard and adopts an imputation view- • Here, the employees did not fall within the category and could therefore be contacted.• see page 701 for the main holding

• RESTRICTIONS ON CONTACT WITH UNREPRESENTED PERSONS • Rule 4.3:

• lawyer shall not imply or state that the lawyer is disinterested• when the lawyer knows or reasonably should know that the unrepresented person misunderstands the

lawyer’s role in the matter, the lawyer shall make reasonable efforts to correct the misunderstanding. The law shall not give an unrep. person legal advice, other than to secure counsel, if the lawyer knows or reasonably should know that the interests of such a person are or have a reasonable possibility of being in conflict with the interests of the client.

• chart on 705• Comment 1: Sometimes a lawyer will have to identify their client to the unrepresented person, and

explain that the client has interests opposed to those of the unrepresented person. • Comment 2: lawyer can negotiate terms of a transaction or settle dispute with an unrepresented

person. So long as the lawyer has explained that the lawyer represents an adverse party and is not representing the person, the lawyer may inform the person of the terms on which the lawyer’s client will enter into an agreement or settle a matter; prepare documents that require that person’s signature and explain that the lawyer’s own view of the meaning of the document or the lawyer’s view of the underlying legal obligations.

• page 705: the major thing is that when you’re talking to an unrepresented person, you can’t leave the impression that their interests are somehow being safeguarded.

• the only advice you can give them is to go get independent counsel.• Problem 12-2: The Complaining Witness:

• Can you go visit her? Yes, under 4.3• what do you have to tell her? you have to make sure she knows you do not represent her, her

interests, etc• make sure you don’t create the misunderstanding that you represent anyone other than the defendant• what about signing a medical release form? You cannot tell her she NEEds to sign it.

• Rule 4.4: respect for Rights of third Persons: • Rule 4.4:

• shall not use means that have no substantial purpose other than to embarrass, delay, or burden a 3d person, or use methods of obtaining evidence that violate the legal rights of such a person

• lawyer who receives a document relating to a client on accident should notify the sender.• all the rules require is that you notify the sender• what else you do with it is a matter of law in your particular jurisdiction. it’s beyond the

scope of the rules...• We don’t want unwarranted intrusion into things that should be private• Comments 1, 2, 3.

Page 55: Professional Responsibility Outline

• Problem 12-3: The Break-in • your client broke into her ex-husband’s house and took documents as proof that he molested their

girls. she photocopied all the documents and brought them to you. what do you do with them?• you didn’t technically assist but it’s not inadvertent b/c you know about it• 8.4• you could try to ask the girls about them to get more• real case: atty did not destroy, just showed the ex-husband’s atty some of the documents. atty said

she knew the ex was a child molester. the ex’s atty was shocked. in court, the ex did not contest full custody. even if they had not reached this settlement, you could go the route of asking the girls to get more information leading to a warrant and uncovering of the evidence.

• the atty never even asked where the documents came from bc he was so shocked at them• DUTY OF PROSECUTORS

• Prosecutors are atty’s so are held under the general rules, as well as addt’l special rules b/c they represent the state

• Footnote on page 713: FN 53• Page 714: Prosecutorial midconduct• UNDERCOVER INVESTIGATIONS

• McDade Amendment: page 716• “an atty for the gvt shall be subject to state laws and rules, and local federal court rules, governing

atty’s in each state where such atty engages in that atty’s duties, to the same exten and in the same manner as other attys in that state.

• The enactment of this amendment however, seems to have had little effect on the ability of federal prosecutors to direct indictment communications with a represented suspect

• Problem 12-4: Prosecutor’s masquerade: • is there an emergency exception to the rules?• Weigh the emergency exception against:

• 5th and 6th amend?• you could try to avoid getting self-incriminating statements.

• Rule 3.8 (it’s really the analog of 3.1) Comment 2: the prosecution should not seek from a defendant that has no representation a waiver of legal rights

• 4.1: false statements• 8.4: false statements• 4.3: are you misleading this client about your role? (misleading an unrepresented suspect)

• Real case: atty went ahead with the plan. the public took the viewpoint that the atty was a hero for doing this- editorials were published, etc. (Denver). There was tremendous public support for impersonating. However, lawyer violated rule 4.3 and 8.4 and was suspended for 3 months. he was required to retake the MPRE, take 20 hours of CLE on ethics. anytime he spoke to an unrepresented person, he had to be accompanied by his supervisor. he appealed but it was affirmed. He wasn’t fired anything, continued to be senior DA.

• REQUIRED INVESTIGATION BY PROSECUTORS BEFORE CHARGES ARE FILED • CONCEALMENT OF ECULPATORY EVIDENCE

• Nifong was disbarred in this case.• UNRELIABLE EVIDENCE

• Unreliable evidence: just b/c something is scientific evidence, our reliance on this may be unreliable.• ENFORCEMENT

• CONDUCT PREJUDICIAL TO THE ADMINISTRATION OF JUSTICE • Problem 12-5:

• Charles Stinson was on a radio show and he made comments about the attorneys who represented detainees at Guantanamo Bay.

• you are on an assistant bar counsel for the state’s bar. what should you do?• STOPPE DHERE, finish chapter 12 and talk bout judicial conduct

• ARE LAWYERS REALLY TOO ZEALOUS?

Page 56: Professional Responsibility Outline

• Judicial canons: • Impartiality is the central theme of all 4 canons.• They define the judge’s role• Canon 1: uphold and promote judicial independence and integrity (Impartiality)

• 1.2 Promoting Confidence- independence, integrity, impartiality • Canon 2: shall perform duties impartially, competently, and diligently

• 2.2 perform all duties fairly and impartially (neutrally)• 2.3 no bias• 2.4 External influences• 2.5 Competence, Diligence, Cooperation• 2.9 Ex Parte• 2.10 Judicial Statements• 2.11 Disqualification

• Canon 3: Minimize Risk of Conflicts• 3.1 Extrajudicial Activities• 3.2 Appearances• 3.3 Testifying as a Character Witness• 3.6 Can’t be apart of discriminatory organization

• Canon 4: political campaigns• 4.1A (12) Announcement Clause• 4.1A (13) Pledge clause• 4.2 Public Elections• 4.4 Campaign Committees

• Exam: don’t need to mention numbers for rules and comments• when can personal conflicts be imputed under rule 1.10?

• Look at comment 3: you’re not going to impute a personal bias to an entire firm (Ex: personal conflict of inability to represent KKK- everyone else at the ACLU is not forbidden). The lawyer just won’t work on the case.

• what if you own the corp that your firm is going to sue? now that is imputed to the entire firm.• 8.3: “Substantial question” should be evaluated by the seriousness of the offense. there is some discretion,

and it is permissive- not mandatory.• Model Rules apply to UNadmitted lawyers• 1.6 and 1.2: when would something be impliedly authorized to reveal confidential? the atty is the client’s

agent- if you needed to know more aboutyour client, you coudl reveal confidences to findout more about your case- do so using a hypothetical. (cmt 4 to 1.6)

• Foreseeability: Restatement comes into play.• crime/fraud: OVerall, the rules try to deal with public harm. 1.6 is really concerned with disclosing

confidential info.• A-C privilege: Look at the content of the information and see how it relates to the AC privilege- certain

facts are going to be well known anyways.• Diminished capacity: is it permissive or mandatory? it’s permissive. “Shall TRY to maintain a normal

relationship”...• The problem with the old lady

• try to make it a normal relationship• don’t impose too much• but eventually you may have to make some determinations (may take reasonably necessary actions)• the rule is designed to TRY to maintain th eatty-client relationship.• Could this lead to a mandatory obligation? Possibly yes, at some point.

• Problem 7-1 my client’s subsidiary:• lawyer cannot represent the company unless they reasonably believe the client would not be effected, but

under the facts of this problem, that’s probably not possible

Page 57: Professional Responsibility Outline

• Informed consent: you should try to get it in writing. this is different if you’re entering transaction with client- then you MUSt have it in writing.

• Problems: • 5-5: p 332: Rule 1.14-

• if reasonably possible, maintain a normal relationship with her. • there may come a time when you can’t, and we would try to direct her (not make her)

• 5-6 can you sue the client? Yes, but that’s not the point of the question. page 334- you should look at getting client release, try to have normal relationship, possibility of terminating relationship 1.16. look at comment 6 to 1.14- all of the things you should take into consideration

• 7-2, 7-3, 7-4:• that problem is baed on the Abner Louiana case in NY: Key text is “Independence” then consider• 7-2: is there a risk that rep would be materially limited? Yes. Attorney being in position of wanting the

contract renewed...• 7-3; can you drop one client ? Hot potato doctrine- courts generally disapprove of that• 7-4: reaches point wherse it may be non-permitable. can they prevail under Strickland standard? Did the

atty sell his client out? is he trying to keep the 10 mil contract? • 9-3 page 553: Looked at whether or not there was authority for a non-literal reading of the rule- 1.8 prohibits

paying living expenses. Comment 10. you really cannot provide assistance. the facts were heart wrenching but too bad. maybe try to get emergency assistance from a program

• 10-2 page 586: USSC ultimately said the judge should have recused himself. The attorney should not join in the motion but you should NOT oppose it either- you’re pursuing justice as an officer of the court.

• 10-3: p 587: Clinton was sanctioned for lying under oath in his deposition• Clinton’s attorney had to disavow some of the statements the president made- that started the ball rolling

• if lawyer has prohibited conflict b/w two current clients:• cmts 2 and 4 to 1.7: generalyl absent consent you can’t

• Screening is required gvt to private and private to gvt. No such screening for private firms (look to local rules). remember the spectrum of confidentality regarding gvt employment. some jurisdictions provide for screening.

• Successive conflicts: when a new lawyer comes into firm• 1.9b1 and b2: You have to overcome a presumption that you have acquired info from former rep that is

adverse to your current client.• “making a prediction”• see comment 3 to rule 1.9: you presume that the atty has information that can be used against the client.

this is a high standard so you have to overcome the presumption• you have to maintain confidences even though its former representation• you cannot UNDO work you’ve done for the client• “same or substantially related matter”